Dermatology MCQ With Explinations [PDF]

  • 0 0 0
  • Gefällt Ihnen dieses papier und der download? Sie können Ihre eigene PDF-Datei in wenigen Minuten kostenlos online veröffentlichen! Anmelden
Datei wird geladen, bitte warten...
Zitiervorschau

Dermatology - Item 1 A 45-year-old man is referred to you by his dermatologist for therapy. He was seen several weeks ago by the dermatologist for evaluation of recurrent purpuric lesions on his legs. He has noted the lesions for the past 3 months, and other than some mild burning, the eruption is asymptomatic. The patient denies other complaints, including arthralgias, myalgias, chest pain, and headaches. His medical history includes a motorcycle accident in 1979 that necessitated several operations. He believes that he was given several transfusions at that time. He denies the use of illicit drugs, except for experimentation with marijuana during college. Physical examination is normal, except for multiple small, palpable, purpuric lesions on his legs. No ulceration is present. Skin biopsy specimen shows a leukocytoclastic vasculitis. Results of laboratory tests include a positive hepatitis C antibody, a positive rheumatoid factor titer of 1:16, and a hepatitis C viral RNA load of 600,000 copies/mL. Results of cryoglobulin testing are negative. A liver biopsy reveals periportal fibrosis with some bridging. The most appropriate therapy for this patient is: (A) Interferon and ribavirin (B) Methotrexate and folic acid (C) Prednisone and azathioprine (D) Cyclophosphamide (E) Intravenous immune globulin Dermatology - Item 2 A 28-year-old woman is evaluated in your office for the recent onset of a pruritic vesicular eruption, which is localized to her thighs and lower legs (Figure). She is otherwise healthy, has no systemic symptoms, and is taking no medications regularly. Except for her skin, the results of the physical examination are normal. See the figure What is the most likely diagnosis? (A) Allergic contact dermatitis (B) Bullous pemphigoid (C) Dermatitis herpetiformis (D) Herpesvirus infection (E) Bullous impetigo Dermatology - Item 3 A 19-year-old man comes to your office for evaluation of acne that is almost exclusively comedonal. Which of the following treatments is most appropriate for this patient? (A) Topical erythromycin (B) Topical benzoyl peroxide (C) Topical tretinoin (D) Systemic tetracycline (E) Systemic isotretinoin

Dermatology - Item 4 A 33-year-old man comes to your office with his two sons because he recently had a “thin” melanoma removed from his back. His physician told him that the “odd-looking” moles scattered on his trunk, arms, and legs predispose him to develop melanoma. The patient would like to know if anyone else in his family has this type of mole. His older son, who is 13 years old, has numerous moles, but the younger son, who is 7 years old, has a few moles (Figure). See the figure What advice would you give the patient about his younger son? (A) The moles should be surgically removed over the next year. (B) Biopsy of one of the lesions should be done to make sure it is normal. (C) His son should be referred to a dermatologist for monitoring with yearly skin examinations. (D) The moles should be frozen with liquid nitrogen to prevent them from progressing. (E) No further management is necessary. Dermatology - Item 5 A 52-year-old man comes to your office for evaluation of a several-year history of intermittent abdominal pain and fatty, foul-smelling stools. He has no nausea or melena, but he has lost approximately 2.3 kg (5 lb) in the last several months. The patient has otherwise been well, but he indicates that he has had a long history of pruritic lesions on the elbows, knees, and buttocks. Physical examination reveals several grouped, crusted lesions on his elbows and knees. Examination of the oral mucosa is normal. There is some right abdominal fullness on palpation. The remainder of the physical examination is normal. What is the most likely diagnosis? (A) Porphyria cutanea tarda (B) Dermatitis herpetiformis (C) Paraneoplastic pemphigus (D) Linear IgA bullous dermatosis (E) Recurrent herpes simplex Dermatology - Item 6 A 67-year-old woman comes to your office for evaluation of a 3-week history of generalized pruritus that wakes her at night. Physical examination shows widespread excoriations and a linear burrow between her thumb and index finger. She mentions that her 3-year-old grandchild, whom she cares for on a daily basis, has similar symptoms. She would like you to write a prescription for both of them. What is the topical treatment of choice for this condition in patients of all ages? (A) Benzyl benzoate (B) Crotamiton (C) Lindane 1% (D) Permethrin 5% (E) 10% sulfur in petrolatum

Dermatology - Item 7 A 34-year-old man is evaluated in your office for a 4-month history of fragility of the skin on the dorsal surface of his hands. He is otherwise healthy. He denies smoking and the use of illicit drugs but admits that he drinks alcoholic beverages daily. He notes that he was abused as a child and remembers having to be cared for in a hospital on multiple occasions, but he does not remember if he received a blood transfusion. He is married and has been monogamous, and he believes that his wife has also been monogamous. Physical examination reveals blisters, erosions, and milia on the dorsum of his hands. There is no hypertrichosis or hyperpigmentation. Results of laboratory tests reveal a normal complete blood count and chemistry panel. Urinary uroporphyrin levels are elevated about 10 times normal. Serum iron level is elevated about 1.5 times normal, as is the total iron-binding capacity. A hepatitis profile reveals a positive hepatitis C antibody, but negative hepatitis A and B antibodies. Liver biopsy reveals fibrosis and bridging of the portal triads, but cirrhosis is not evident.

In addition to advising that he stop drinking alcohol, the most appropriate initial therapy for this patient is: (A) Interferon (B) Phlebotomy (C) Ribavirin (D) Sodium bicarbonate Dermatology - Item 8 A 31-year-old man comes to your office for evaluation of a vesicular eruption on the instep of his left foot. Potassium hydroxide preparation of the lesion shows septate hyphae. What is the most likely diagnosis? (A) Candidiasis (B) Tinea versicolor (C) Dermatophytosis (D) Herpesvirus infection (E) Impetigo Dermatology - Item 9 A 19-year-old woman with severe, therapy-resistant acne with early scarring is evaluated in your office because her acne has not improved with oral antibiotic therapy. You are considering treatment with oral isotretinoin. Which of the following laboratory assessments is the most crucial to monitor regularly if oral isotretinoin therapy is initiated in this patient? (A) Complete blood count (B) Liver function tests (C) Cholesterol level (D) Urinalysis (E) Pregnancy test

Dermatology - Item 10 A 19-year-old cross-country runner comes to your office because she needs a physical examination for clearance to participate on the varsity track and field team. During the course of the examination, you note that she has brown hair, brown eyes, a medium complexion, and more than 100 small (2- to 4-mm), relatively flat to slightly elevated, well-demarcated, evenly pigmented, dark moles scattered over her trunk and extremities (Figure). She says that she tans easily and deeply. She can recollect having a sunburn only one time, the first day of a vacation in Hawaii with her parents when she was 12 years old. There is no family history of melanoma. See the figure What should you tell this patient about her moles? (A) Her moles are normal and of no significance. (B) She is in grave danger of developing a melanoma before age 30 years. (C) Although the moles present no danger to her, her children will be at increased risk of melanoma. (D) Her children will be at increased risk for congenital nevi. (E) Her moles increase her chances of developing a melanoma. Dermatology - Item 11 Which one of the following helps distinguish herpes simplex virus infection from herpes zoster virus infection? (A) Pruritus (B) Periodic recurrence in the same area (C) Prodromal neuralgia (D) Multiple vesicles Dermatology - Item 12 A 37-year-old man comes to your office for a physical examination. On examination, he has light-brown hair, brown eyes, and only a few scattered moles, none of which is atypical. However, he has a melanoma on his upper back. Except for some freckle-like lesions across his shoulders and upper back, his skin does not seem to be photodamaged or weathered. The patient reports that he does not have many outdoor pursuits, other than coaching the boys’ soccer team after school. He says he tans if he spends enough time outdoors. What history would best explain the development of this patient’s melanoma? (A) A series of five tanning booth sessions (B) Periodic weekend hiking trips (C) Multiple childhood sunburns (D) Chronic exposure to radiation from indoor fluorescent lighting (E) A 2-week scuba diving trip last summer

Dermatology - Item 13 A 65-year-old woman comes to your office for evaluation of severe central facial telangiectasias. The differential diagnoses include rosacea, systemic lupus erythematosus, and dermatomyositis. Which of the following is the best test to determine the diagnosis? (A) Direct immunofluorescence microscopy (B) Skin biopsy for routine processing (C) Antinuclear antibody test (D) Complete blood count (E) Serum creatine kinase level Dermatology - Item 14 A 68-year-old man is evaluated in your office for a 3-week history of pruritic and slightly painful bullae on his back, chest, abdomen, and thighs. He was initially treated by his primary care physician with oral dicloxacillin for possible bullous impetigo. He subsequently developed additional blisters and urticarial papules. On physical examination, multiple tense blisters and some flaccid blisters are noted on the trunk and proximal lower extremities, along with several edematous, red papules (Figure). There are no oral lesions or lesions on his scalp or arms. See the figure What should be done to immediately determine the level of the pathologic process in the skin? (A) Refer the patient to a dermatologist, and request a skin biopsy for routine processing and direct immunofluorescence. (B) Perform a Tzanck test. (C) Push on an intact blister to see if it can be extended. (D) Rub an edematous papule to see if it urticates. (E) Unroof a blister, and culture the area for atypical mycobacteria. Dermatology - Item 15 A 64-year-old man comes to your office for evaluation of lesions on his hands (Figure). See the figure What is the most likely diagnosis? (A) Gout (B) Psoriasis (C) Dermatophytosis/onychomycosis (D) Rheumatoid arthritis (E) Sarcoidosis

Dermatology - Item 16 A 30-year-old man presents to your office because of central facial flushing in association with his rosacea. Which of the following is the most important for this patient to avoid? (A) Alcohol (B) Caffeine (C) Spicy foods (D) Sunlight (E) Hot beverages Dermatology - Item 17 A 28-year-old woman with a history of atopic dermatitis is hospitalized because of a tender, red, swollen left hand. On physical examination, she has a temperature of 38.3 °C (101 °F) and enlarged, tender left axillary lymph nodes. Intravenous vancomycin therapy is initiated. Five days after initiation of therapy, she develops several tense, sausage-shaped blisters, some in an annular configuration, on the trunk, which cannot be extended with application of pressure. What is the most likely diagnosis? (A) Bullous impetigo (B) Drug-induced linear IgA bullous dermatosis (C) Acute contact dermatitis (D) Acute flare of atopic dermatitis (E) Pemphigus Dermatology - Item 18 A 30-year-old homosexual man comes to your office for evaluation of numerous asymptomatic skin lesions on his face and neck. He had a partner who died several years ago of Pneumocystis carinii pneumonia. He knew that his partner was HIV-positive, but he has refused to be tested for HIV infection until now. Physical examination reveals multiple facial lesions, as well as some lesions on his neck. He also has a scaly rash involving the nasolabial folds, eyebrows, and scalp. Results of laboratory tests include a positive HIV antibody and a CD4 count of 50/µL. Several months following the initiation of highly active antiretroviral therapy (HAART), the CD4 count increases to 500/µL, and the viral load is undetectable. The patient’s skin lesions are likely to: (A) Remain stable (B) Slowly progress (C) Resolve (D) Become secondarily infected (E) Transform into a malignancy

Dermatology - Item 19 A 24-year-old man is evaluated in your office for a 6-month history of painful oral ulcers and crusted skin lesions. He believes that the process began following a tick bite, but he was never able to find the tick. He is a cement worker and had been off work recently because of an injury to his hand. Since returning to work, he is fatigued and has lost 2.7 kg (6 lb) without trying to lose weight. He denies other health problems. He is taking no medications and does not smoke or chew tobacco. He denies intravenous drug use but has smoked marijuana on occasion and drinks four to six beers daily after work. On physical examination, vital signs are normal. There are ulcerations bilaterally on his buccal mucosa (Figure) and crusted verrucous lesions on his face and upper back. Biopsy of the lesions in the mouth and on the back is done and reveals suprabasilar clefting with acantholysis. Immunofluorescence microscopy of specimens from the mouth and adjacent normal skin reveals IgG deposition within the epidermis. Indirect immunofluorescence of serum using monkey esophagus and rat bladder as substrate reveals intercellular antibodies. See the figure The most likely diagnosis is: (A) Impetigo (B) Cutaneous lupus erythematosus (C) Sweet’s syndrome (D) Paraneoplastic pemphigus (E) Glucagonoma syndrome Dermatology - Item 20 An 80-year-old man with a history of type 1 diabetes mellitus, hypertension, and coronary artery disease comes to your office for evaluation of several 0.5- to 1.0-cm bullae on the abdomen and thighs. A skin biopsy is done for routine processing and direct immunofluorescence, and results show a subepidermal blister with numerous eosinophils and the linear deposition of C3a and IgG at the basement membrane zone, respectively. What is the most appropriate treatment for this patient? (A) Tetracycline and niacinamide (B) Prednisone (C) Dicloxacillin and topical clobetasol (D) Azathioprine and prednisone (E) Methotrexate Dermatology - Item 21 A 64-year-old man comes to your office for evaluation of bruises on his face that have appeared over the past 12 months. He has been thought to be in good health but recently has noted fatigue and mild dyspnea with exercise. He has noticed that the bruising is worse after episodes of constipation, when he has had to strain to defecate. On physical examination, vital signs are normal. Several macular, purpuric areas are noted on the patient’s face (Figure). The tongue may be slightly enlarged. Cardiac examination reveals a mid to late systolic murmur, but no extra sounds are heard. A skin biopsy reveals eosinophilic, amorphous material around the dermal blood vessels. Serum protein electrophoresis reveals a monoclonal paraprotein. See the figure The most likely diagnosis is:

(A) Actinic purpura (B) Primary systemic amyloidosis (C) Idiopathic thrombocytopenic purpura (D) Small-vessel vasculitis (E) Kaposi’s sarcoma Dermatology - Item 22 A 24-year-old woman is evaluated in your office because she has had urinary frequency and burning for the past 3 days. Urinalysis shows numerous leukocytes and erythrocytes, and a urine sample is sent for culture. Therapy with double-strength trimethoprimsulfamethoxazole, twice daily, is started. Three days after initiation of therapy, the patient develops generalized malaise and a diffuse redness and tenderness to her skin. On physical examination, she has a generalized erythema on her face, upper chest, back, abdomen, arms, and proximal lower extremities. Several bullae are noted on the abdomen, and two erosions are visible on the buccal mucosa. What is the most appropriate treatment for this patient? (A) Continue trimethoprim-sulfamethoxazole, and begin prednisone, 40 mg/d. (B) Discontinue trimethoprim-sulfamethoxazole, and start topical triamcinolone cream, 0.1% twice daily, to the affected areas. (C) Discontinue trimethoprim-sulfamethoxazole, and begin prednisone, 25 mg/d. (D) Discontinue trimethoprim-sulfamethoxazole, and hospitalize the patient for intravenous immunoglobulin therapy. (E) Discontinue trimethoprim-sulfamethoxazole, and hospitalize the patient for treatment with intravenous gentamicin and ampicillin. Dermatology - Item 23 A healthy 40-year-old nurse comes to your office for evaluation of a 1-month history of vesicular eruptions on the dorsum and the distal portion of her hands. What is the best initial topical therapy for this patient? (A) Calcipotriene (B) Neosporin (C) Clotrimazole-betamethasone dipropionate (D) Triamcinolone acetonide (E) Tretinoin

Dermatology - Item 24

A 58-year-old woman who recently underwent wide local excision for a melanoma of the right thigh comes to your office for a follow-up visit. She also had sentinel node examination of the ipsilateral groin nodal basin. The sentinel node was negative for tumor. The melanoma measured 1.9 mm in thickness and was classified as a Clark level IV lesion. As her primary care physician, you plan to see the patient at 3-month intervals. What is the most useful test for detecting early metastases during this patient’s followup visits? (A) Serial serum lactate dehydrogenase levels (B) Biannual chest radiography (C) History and physical examination (D) MRI of the brain (E) CT of the thorax, abdomen, and pelvis Dermatology - Item 25 A variant of scabies, “crusted scabies,” is frequently associated with which of the following underlying conditions? (A) Acute myelogenous leukemia (B) Cystic fibrosis (C) Epstein-Barr virus infection (D) HIV infection (E) Psoriasis Dermatology - Item 26 A 38-year-old woman comes to your office for evaluation of a 4-month history of a slowly progressive papular and pustular erythematous eruption on both cheeks (Figure). There is no history of skin disease. The application of fluorinated glucocorticoid cream produced transient improvement, followed by pronounced worsening. See the figure What is the most likely diagnosis? (A) Dermatophytosis (B) Contact dermatitis (C) Systemic lupus erythematosus (D) Rosacea (E) Seborrheic dermatitis

Dermatology - Item 27

A 42-year-old woman comes to your office for a follow-up visit after removal of a melanoma from her upper thigh 1 month ago. She has spent a great deal of time suntanning all her life, and she has suffered sunburns at the beginning of each summer since her teens. Her skin is weathered, with numerous freckle-like lesions on her shoulders and upper back. The tumor she had removed 1 month ago had clear margins and a 0.8-mm Breslow thickness and was classified as a Clark level III lesion. You plan to see her for follow-up examinations every 3 months for the next year and twice a year after that. In addition to supplying her with sun-protection program literature, what else should be done in managing this patient? (A) Teach her how to do monthly skin self-examinations. (B) Start her on a course of monthly intramuscular interferon injections. (C) Do immediate lymphoscintigraphy and sentinel node biopsy. (D) Refer her for psychologic counseling for high-risk behavior modification. (E) Do biannual CT scans of the skull, chest, and abdomen. Dermatology - Item 28 A 52-year-old woman is evaluated in your office for a 4-year history of a recurrent rash that occurs three to four times a year on her buttocks. She often notes a slight “tingling” feeling before the eruption occurs. She describes the acute eruption as a flat, red, slightly tender area. No blisters ever develop on the site. Although she has no acute eruption at present, there is a well-defined, nonscaling, brown-gray macule (measuring 3.0 cm ´ 5.2 cm) on the left lateral gluteal area. The patient does not apply topical agents on the area. Her only medications are an occasional aspirin and laxative as needed. What is the most likely diagnosis? (A) Recurrent herpes simplex virus infection (B) Recurrent contact dermatitis (C) Recurrent urticaria (D) A fungal infection (E) A fixed-drug eruption Dermatology - Item 29 A 20-year-old man comes to your office for evaluation of acute contact dermatitis from poison ivy (rhus dermatitis) on his left and right forearms. He also has chronic psoriasis with plaques on the knees, elbows, scalp, and trunk. Which of the following is the best treatment for this patient? (A) Topical clobetasol (B) Oral prednisone in a 2-week taper (C) Oral methylprednisolone dose pack (D) Intramuscular triamcinolone acetonide (E) Loratadine Dermatology - Item 30

A 65-year-old man presents with a lesion on his cheek (Figure). The patient says it has been slowly enlarging for about 10 years and has never bothered him in any way. See the figure What is the appropriate management for this patient? (A) Refer him to a dermatologist for a biopsy of the lesion. (B) Remove the lesion with liquid nitrogen. (C) Remove the lesion with laser treatment. (D) Perform appropriate laboratory tests. (E) No management is necessary. Dermatology - Item 31 A 67-year-old man with hypertension, mild heart failure, bipolar disorder, rosacea, and psoriasis comes to your office for evaluation. The psoriasis was previously stable and confined to his elbows and knees, but it has suddenly become widespread. Which one of the following drugs being taken by the patient is likely to have exacerbated his psoriasis? (A) Hydralazine (B) Lithium (C) Hydrochlorothiazide (D) Digoxin (E) Tetracycline Dermatology - Item 32 A 45-year-old woman comes to your office for evaluation of a rash that has been present for the past month. Recently, she has noted a marked amount of fatigue and has had difficulty maintaining her normal activities, including a regular tennis game. She states that her legs feel heavy after climbing one flight of stairs in her house and that she has had difficulty drying her hair with a hair dryer. She denies other problems, including weight loss, and a review of systems is unrevealing. Physical examination shows erythematous, scaly plaques on the dorsal surface of the hands; a poikiloderma on the upper chest (Figure); an edematous, violaceous eruption around the eyes; and weakness in the shoulder and pelvic girdle muscles, but normal grip strength. The remainder of the physical examination is normal. See the figure The test most likely to detect a malignancy in this patient is: (A) Barium enema (B) Brain scan (C) Carcinoembryonic antigen (CEA) level (D) Pelvic ultrasonography (E) Thyroid scan Dermatology - Item 33

A 22-year-old woman is started on empiric phenytoin therapy after undergoing a craniotomy for a subdural hematoma resulting from an automobile accident. Twelve days later, she develops a temperature of 38.3 °C (101 °F), and therapy with intravenous ceftazidime is started after the results of blood cultures are received. Two days later, the patient develops a generalized, erythematous, papular eruption. A complete blood count shows a leukocyte count of 12,800/µL, with 68% neutrophils, 2% bands, 9% eosinophils, 3% basophils, and 17% lymphocytes. Liver function tests reveal a serum bilirubin level of 1.2 mg/dL, aspartate aminotransferase level of 66 U/L, and an alanine aminotransferase level of 72 U/L. Her serum electrolyte, blood urea nitrogen, and serum creatinine levels are normal. What should be done next for this patient? (A) Discontinue both medications, and treat with intravenous methylprednisolone sodium succinate. (B) Discontinue phenytoin, and continue ceftazidime. (C) Substitute carbamazepine for phenytoin. (D) Continue phenytoin and ceftazidime. (E) Continue phenytoin, and discontinue ceftazidime. Dermatology - Item 34 A 59-year-old man is referred to your office for evaluation of a pigmented lesion on his chest (Figure), which he has had for several months. The lesion occasionally becomes inflamed and tender. See the figure What is the most likely diagnosis? (A) Congenital nevus (B) Malignant melanoma (C) Pigmented basal cell carcinoma (D) Seborrheic keratosis (E) Solar lentigo Dermatology - Item 35 A 48-year-old woman comes to your office for evaluation of an eruption on her face, arms, and upper chest. She first noticed the eruption about a month ago while she was on vacation in Hawaii. She has a history of having rashes following intense sun exposure, but in the past, they have always faded within several days to 1 week. This time, her rash not only has persisted but also has spread. She also complains of fatigue and stiffness in the morning. Physical examination reveals a papular eruption on her cheeks and annular erythematous lesions on the upper extremities (Figure). There are no lesions on her ears or scalp. The remainder of the physical examination is normal. See the figure The test most likely to confirm the diagnosis of this patient’s condition is:

(A) Skin biopsy for routine processing

(B) Immunofluorescence microscopy (C) Antinuclear antibody (D) Anti-Ro (anti–SS-A) antibody (E) Schirmer’s test Dermatology - Item 36 A 65-year-old woman is referred to your office because of sudden pancytopenia, nausea, and elevated liver enzyme levels. She has been taking weekly low-dose pulse methotrexate therapy for severe psoriasis for 3 years and has had stable hematologic and liver function parameters. Which medication recently added to this patient’s regimen most likely accounts for her methotrexate toxicity? (A) Itraconazole (B) Prednisone (C) Propranolol (D) Trimethoprim-sulfamethoxazole (E) Minocycline Dermatology - Item 37 A 73-year-old man has recently moved to your city and is consulting you for the first time about his skin. He has lived in the southwestern United States his entire life and plays golf regularly since retiring 11 years ago. On physical examination, the patient’s skin is heavily sun-damaged. He has many actinic keratoses on his face, ears, forearms, and the dorsum of his hands. No lesions of skin cancer are apparent. Which of the following would be the best management strategy for this patient? (A) Treat the thicker actinic keratoses with cryotherapy (liquid nitrogen). (B) Apply topical 5-fluorouracil (5-FU) cream, twice daily for 2 weeks. (C) Refer the patient for photodynamic therapy. (D) Treat individual lesions with liquid nitrogen, followed by a course of topical 5-FU cream. (E) Reassure the patient, and re-examine him in 6 months.

Dermatology - Item 38

A 60-year-old woman is evaluated in your office for a rash on her upper chest and arms. The rash has been present for about 2 months. She notices that it worsens following her weekend golf game. She has a history of hypertension and is postmenopausal. She has had problems with intermittent “acne” on her face since early adulthood. Her medications include hydralazine and hydrochlorothiazide for her hypertension, minocycline for her acne, and conjugated estrogens and etidronate to prevent osteoporosis. Her medical history includes a blood transfusion following childbirth. She has had four pregnancies that resulted in three healthy children. Her one pregnancy loss occurred as a first-trimester spontaneous abortion. She smokes 1 pack of cigarettes daily and drinks an occasional mixed drink on the weekends when she is out with friends at a restaurant. She states that she has never used illicit drugs. There is no significant family history, except for allergic rhinitis in several of her siblings. Physical examination reveals an annular erythematous eruption on the chest and arms (Figure). She has mild peripheral edema. Her blood pressure is normal. The remainder of her physical examination is normal. Results of laboratory tests include a normal complete blood count, urinalysis, and chemistry panel. A skin biopsy reveals an interface dermatitis compatible with a diagnosis of cutaneous lupus erythematosus. She has a positive antinuclear antibody > 1:640, speckled; positive anti-Ro (anti-SS-A) and anti-La (anti-SS-B) antibodies; and a negative anti-nDNA antibody. See the figure The most likely cause of this patient’s eruption is: (A) Conjugated estrogens (B) Hydralazine (C) Hydrochlorothiazide (D) Minocycline (E) Etidronate Dermatology - Item 39 You advise the patient in Item 38 to stop smoking, but her condition continues to be unaffected by treatment. Which of the following therapies should now be prescribed? (A) Intravenous immunoglobulin (B) Carbamazepine (C) Mycophenolate mofetil (D) Thalidomide (E) Prednisone Dermatology - Item 40 A 72-year-old man comes to your office because of a 1-week history of severe oral sores and a rash on his trunk and upper arms. He notes that he has been more tired in the last 2 months and has lost approximately 2.3 kg (5 lb), which he attributes to his age. His only medication is a thiazide diuretic, which he has been taking for the last 8 years for mild hypertension.

On physical examination, multiple oral erosions are present on the buccal mucosa and tongue. There are numerous edematous papules and bullae (1.0 to 1.5 cm in diameter) on the trunk. Pressure applied to the edge of one of the blisters results in its extension. What is the most likely diagnosis?

(A) Paraneoplastic pemphigus with autoantibodies against type VII collagen (B) Severe, erosive lichen planus with no autoantibodies (C) Pemphigus vulgaris or paraneoplastic pemphigus with antibodies against desmoglein 3 or desmoglein 1 and 3 and the plakin proteins, respectively (D) Bullous pemphigoid with antibodies against bullous pemphigoid antigen 1 (BPAG1) and BPAG2 (E) A thiazide-induced blistering reaction with detectable IgE antibodies against desmoglein 1 and the plakin proteins Dermatology - Item 41 A 57-year-old man comes to your office for evaluation of a hairy mole on his left sideburn. He reports that he was born with this mole and that it has undergone some changes since he saw you last year. The hairs in it, which have always been dark, have turned gray, and a small bump has appeared. It has not bled, itched, or hurt. The lesion measures approximately 4 ´ 2 cm (Figure). See the figure What is the next step in the management of this patient? (A) Reassure the patient that these are normal aging changes of a congenital nevus. (B) Perform a shave (tangential) biopsy of the nodule to rule out melanoma. (C) Arrange for excision of the entire lesion. (D) Defer biopsy until lymphoscintigraphy can be done. (E) Examine the lesion at 3-month intervals for further changes. Dermatology - Item 42 A 27-year-old man has developed a generalized pruritic eruption (Figure) 1 year after receiving an allogeneic bone marrow transplantation. See the figure Which of the following dermatologic conditions most closely resembles this characteristic eruption of chronic graft-versus-host disease? (A) Lichen planus (B) Pityriasis rosea (C) Psoriasis (D) Atopic dermatitis (E) Tinea corporis Dermatology - Item 43 A 21-year-old woman received a prescription for a medication from her primary care physician who warned her about avoiding sun exposure while taking the medication. However, she did not reduce her time of exposure in her tanning bed, and she suffered a severe burn.

Which drug is the most likely offending agent? (A) Colchicine

(B) Hydroxychloroquine (C) b-Carotene (D) Doxycycline (E) Azathioprine Dermatology - Item 44 A 20-year-old man comes to your office for evaluation of a 1-week history of pruritic papules. The lesions last from 3 to 5 hours and then resolve. He says that the eruptions are associated with playing basketball or lifting weights, and the lesions often occur at the sites of trauma. He has a history of atopic dermatitis and allergic rhinitis, but these conditions have been inactive for the last 2 years. He is not taking any medications and does not have recurrent fever sores. On physical examination, there are no visible cutaneous lesions; however, when the skin is stroked, an immediate wheal and flare reaction develops. What is the most likely diagnosis?

(A) Pressure-induced urticaria (B) Urticarial vasculitis (C) Cholinergic urticaria (D) The urticarial form of erythema multiforme (E) Symptomatic dermographism Dermatology - Item 45 A 23-year-old professional golfer comes to see you because he would like to know what he can do to prevent skin cancer. The patient has a light complexion and red hair. What advice would you give this patient? (A) Avoid the sun when possible. (B) Rely primarily on high-SPF sunscreens. (C) Rely primarily on protective clothing and hats. (D) Keep a “protective” tan all year by going to a tanning salon. (E) Limit tournaments to locations in northern latitudes. Dermatology - Item 46 A 48-year-old avid fisherman comes to your office to ask your advice about appropriate sun-protective hats. Until now he has worn a small thin-brimmed fishing cap. His ears have become burned on occasion, and his neck shows the effects of chronic sun exposure. What recommendation would you give to this patient? (A) Avoid hats altogether, and use sunscreen on the face, neck, and ears. (B) Wear a broad-brimmed, fenestrated straw hat to keep cool. (C) Wear a 4-inch broad-brimmed cloth hat. (D) Wear a baseball cap with a 4-inch brim. (E) Wear a baseball cap with removable earflaps and neck flaps.

Dermatology - Item 47 Which of the following clinical features is most characteristic of chronic eczematous eruptions regardless of the underlying disease (for example, contact dermatitis, stasis dermatitis, or atopic dermatitis)? (A) Vesicles (B) Lichenification (C) Koebner phenomenon (D) Dermal edema (E) Annular configuration Dermatology - Item 48 A 63-year-old man, who is a retired forest ranger, comes to your office for evaluation of numerous rough, scaly spots on the dorsum of his hands and on his forearms (Figure). Before you outline a management plan, you would like him to disrobe for a total skin examination in order to check for more serious skin cancer. See the figure What malignancy would most likely be found in this patient? (A) Keratoacanthoma (B) Basal cell carcinoma (C) Melanoma (D) Squamous cell carcinoma Dermatology - Item 49 A 45-year-woman whom you have been seeing intermittently over the years for asthma is planning a mid-winter Mediterranean cruise and wants to know about the wisdom of obtaining a “base tan” at a tanning salon. She plans to soak up as much sun as she can on this once-ina-lifetime vacation and wants to be prepared the minute the ship sails. She claims she can get a tan if she works at it but will often burn during her first outing at the beach each summer. You note she has light brown hair and eyes and a light complexion. There is some fine wrinkling of her face, but no characteristics associated with photoaging. What recommendation would you give to this patient? (A) By all means get the “base tan.” (B) Avoid going to the tanning salon entirely. (C) Go for a few tanning salon sessions, but plan to use sunscreen on board the ship. (D) Continue tanning salon sessions on her return to sustain her tan. (E) Combine tanning salon sessions with as much outdoor activity as possible.

Dermatology - Item 50

A 31-year-old nurse comes to your office for evaluation of a hand rash that she has had for the last 2 years. She suspects that her hand cream is causing the rash, so she recently has changed brands. She wears latex gloves intermittently while caring for her patients. She denies symptoms of allergic rhinitis or asthma. On physical examination, there is a slightly red, scaling, macular eruption on the dorsum of both hands, with a distinctive line at the wrist, but her palms are normal. What is the best method for evaluating this patient’s disorder? (A) Send a serum sample for radioallergosorbent test (RAST) to latex antigen. (B) Obtain a skin biopsy to rule out latex allergy. (C) Refer to an allergist for skin-prick testing to a number of common antigens, including latex. (D) Refer her to a dermatologist for patch testing to a number of common antigens, including rubber additives. (E) Perform an inhalation challenge with latex antigen and pulmonary function tests. Dermatology - Item 51 A 19-year-old woman who works as a lifeguard asks your advice about the sunscreen she has been using. She says that on occasion, the SPF-15 sunscreen she has used in the past has not protected her from getting sunburned after a day at the pool. She admits that she applies it only after arriving at the community swimming pool and has never reapplied it after the initial application. What advice should you give this patient about sunscreens? (A) A sunscreen with an SPF greater than 15 is unnecessary. (B) Sunscreens should be applied about a half hour before going into the sun. (C) Reapplication of sunscreen is required only after swimming. (D) Her redness is a reaction to the sunscreen and not a sunburn. Dermatology - Item 52 A 52-year-old woman who recently moved to Arizona from New York comes to your office for advice on sun-protective clothing. She would like to know if there are any differences in the properties of clothing in hiking shirts. She cannot recall ever having a sunburn, and she tans easily. She is unfamiliar with the type of clothing that would be best suited for extended periods of sun exposure under conditions that may result in heavy sweating, such as hiking. Which of the following characteristics of clothing is most effective in protecting the skin from the sun? (A) Color (B) Tightness of the weave of the fabric (C) Whether or not the garment is wet or dry (D) Thickness of the fabric (E) Whether the garment fits snugly or drapes loosely Dermatology - Item 53

All of the following skin changes seen in older white patients are attributable to photodamage

EXCEPT: (A) Fine wrinkling (B) Lentigines (C) Laxity (D) Red, scaly keratoses (E) Telangiectasias

answer Item 1 Answer: (A) Interferon and ribavirin Educational Objective: Select the appropriate therapy for hepatitis C–associated vasculitis. This patient with hepatitis C–associated vasculitis probably has essential mixed cryoglobulinemia, despite the negative results of cryoglobulin testing. The best therapy for this patient’s vasculitis and early changes of cirrhosis is interferon and ribavirin. Although vasculitis may be treated with prednisone, methotrexate, azathioprine, and cyclophosphamide, the use of these drugs may increase the viral load and result in progression of the liver disease. Methotrexate has been used for some patients with smallvessel vasculitis, but it has been associated with the possible development of cirrhosis and is certainly not indicated in this patient, even with folic acid. Cyclophosphamide and intravenous immunoglobulin are therapies that are far too aggressive for small-vessel vasculitis. Bibliography 1.Reichard O, Norkrans G, Fryden A, Braconier JH, Sonnerborg A, Weiland O. Randomised, double-blind, placebo-controlled trial of interferon alpha-2b with and without ribavirin for chronic hepatitis C. The Swedish Study Group. Lancet. 1998;351:83-7. 2.McHutchison JG, Gordon SC, Schiff ER, Shiffman ML, Lee WM, Rustgi VK, et al. Interferon alfa-2b alone or in combination with ribavirin as initial treatment for chronic hepatitis C. Hepatitis Interventional Therapy Group. N Engl J Med. 1998;339:1485-92. Item 2 Answer: (A) Allergic contact dermatitis Educational Objective: Recognize the clinical manifestations of an acute allergic contact dermatitis, and differentiate it from other acute vesiculobullous diseases. This patient’s tense vesicles and bullae on a background of erythema are typical of a contact dermatitis. In some areas, the eruption assumes a linear configuration, characteristic of an exogenous exposure to an antigen or irritant. Allergic contact dermatitis is almost always pruritic. Biopsy of a lesion shows spongiosis with intraepidermal vesicle formation. Bullous pemphigoid is an autoimmune disease of elderly patients that is characterized by a pruritic eruption, which usually develops gradually. The lesions are tense bullae on erythematous or normal-appearing skin. The common sites of involvement are the groin, axillae, and flexural areas. The blisters are subepidermal in location and are accompanied by an eosinophilic infiltrate. Direct and indirect immunofluorescent studies show a linear band of IgG and C3 deposited along the basement membrane zone. Dermatitis herpetiformis is a chronic, intensely pruritic disorder that usually occurs in adults and is characterized clinically by clustered small vesicles symmetrically located on the extensor surfaces, particularly the elbows, knees, and buttocks. The vesicles are subepidermal, with a neutrophilic infiltrate in the upper dermis. Direct immunofluorescent studies show granular deposition of IgA along the basement membrane zone.

Herpesvirus infection is typically localized to a small area (herpes simplex virus infection) or a dermatome (herpes zoster infection), and it is not widespread and bilaterally distributed like the lesions in this patient. Impetigo/cellulitis usually does not occur in healthy adults. It is rarely widespread and bilateral, and it is characterized by pustules (impetigo) or confluent edematous erythema (cellulitis). Bibliography 1.Leung DY, Diaz LA, DeLeo V, Soter NA. Allergic and immunologic skin disorders. JAMA. 1997;278:1914-23. 2.Belsito DV. A sherlockian approach to contact dermatitis. Dermatol Clin. 1999;17:705-13. 3.Krasteva M, Kehren J, Ducluzeau MT, Sayag M, Cacciapuoti M, Akiba H, et al. Contact dermatitis I. Pathophysiology of contact sensitivity. Eur J Dermatol. 1999;9:65-77.

4.Krasteva M, Kehren J, Sayag M, Ducluzeau MT, Dupuis M, Kanitakis J, Nicolas JF. Contact dermatitis II. Clinical aspects and diagnosis. Eur J Dermatol. 1999;9:144-59.

Item 3 Answer: (C) Topical tretinoin Educational Objective: Select the best therapeutic approach for comedonal acne. Treatment for acne is best directed at its various components. Patients with comedones, which are the dominant feature in this patient, benefit from therapy with a topical retinoid, such as tretinoin. Tretinoin in a microsphere delivery system, adapalene, and tazarotene are newer retinoid alternatives. Normalization of follicular keratinization follows retinoid use, which results in extrusion of plugged sebaceous material. Topical erythromycin is an appropriate therapy for patients with mild to moderate inflammatory acne, but it has little effect on comedones. Topical benzoyl peroxide is also primarily beneficial for patients with mild inflammatory lesions, but the effect on comedones would be far less than that achieved with a topical retinoid. Systemic tetracycline is best used for patients with more significant inflammatory acne. Systemic therapy with oral isotretinoin is generally reserved for patients with the most severe forms of acne that are resistant to traditional therapies. Bibliography 1.Thiboutot DM. An overview of acne and its treatment. Cutis. 1996;57(1 Suppl):8-12. Item 4 Answer: (C) His son should be referred to a dermatologist for monitoring with yearly skin examinations. Educational Objective: Choose the best management strategy for patients with dysplastic nevi. Despite the normal appearance of the 7-year-old son’s moles, he has the dysplastic nevus syndrome. In the family setting described, it is usual for patients with the syndrome to develop moles at an earlier age than would be expected for common acquired nevi. In patients with this syndrome, moles not only appear early in life, but also occur in increased numbers and in locations not ordinarily seen with common acquired moles, such as the scalp, genital region, dorsum of the feet, and breasts (in women). Although the appearance and size of these lesions may be clinically indistinguishable from those of routine moles at this age, the morphology usually resembles that of classic dysplastic nevi when the patient reaches late adolescence. The most appropriate course of action is to advise the patient that his child is afflicted with the dysplastic nevus syndrome and to educate the parent about the importance of having the child avoid sun exposure, as well as about the deleterious effects of sunburns. Yearly complete skin examinations are recommended. If none of the moles is clinically suspicious for melanoma, a biopsy is not indicated. The diagnosis of familial dysplastic nevus syndrome is a clinical one, and, in the setting described, there is no question about the diagnosis. Similarly, there is no compelling need to remove the existing normal-appearing moles in this youngster. It is important that he start a lifetime sun-protection program and have suspicious lesions removed, but wholesale removal is not warranted. Freezing any kind of mole is inappropriate treatment. Bibliography 1.Brodkin RH, Altman EM. Controlling malignant melanoma. A focus on pediatricians. Am J Dis Child. 1993;147:875-81. 2.Halpern AC, Guerry D 4th, Elder DE, Trock B, Synnestvedt M, Humphreys T. Natural history of dysplastic nevi. J Am Acad Dermatol. 1993;29:51-7. Item 5 Answer: (B) Dermatitis herpetiformis Educational Objective: Correlate the clinical findings with the specific blistering disorder. The clinical findings of pruritic lesions on the elbows and buttocks associated with chronic gastrointestinal symptoms strongly suggest the diagnosis of dermatitis herpetiformis. Frequently, these vesicles rupture from local scratching and may appear only as grouped crusts. There is a strong association of dermatitis herpetiformis with gluten-sensitive enteropathy and malabsorption, which explain this patient’s chronic gastrointestinal symptoms. The lesions of porphyria cutanea tarda and linear IgA bullous dermatosis are not

pruritic, and these disorders are not associated with gastrointestinal disease. Although the diagnosis of paraneoplastic pemphigus could be possible in a patient with weight loss and gastrointestinal symptoms, these patients usually have significant oral erosions and nonpruritic polymorphic skin lesions. Recurrent herpes simplex lesions are grouped, but they would not be expected to be pruritic or symmetrical. Bibliography 1.Reunala T, Collin P. Diseases associated with dermatitis herpetiformis. Br J Dermatol. 1997;136:315-8. Item 6 Answer: (D) Permethrin 5% Educational Objective: Prescribe appropriate therapy for patients with scabies. Permethrin 5% and lindane are the mainstays of therapy for patients with scabies. When used properly, both agents are safe, but lindane has the greatest potential for adverse reactions if it is not used properly. Toxicity from lindane poisoning usually takes the form of seizures. Concerns about the neurotoxicity of lindane, particularly in children and in patients who are prone to seizures, have brought permethrin to the therapeutic forefront for scabies, and it is currently the treatment of choice for all age groups. Alternative scabicides include crotamiton 10%, and 6% to 10% sulfur ointments. These preparations are considerably less effective, and they must be applied on a daily basis for 5 to 10 days. The messiness and offensive odor of sulfur also limit patient compliance. Benzyl benzoate has been widely used to treat scabies in developing countries, especially in animals, but it is rarely used in the United States because it is not as effective as permethrin. Bibliography 1.Schultz MW, Gomez M, Hansen RC, Mills J, Menter A, Rodgers H, et al. Comparative study of 5% permethrin cream and 1% lindane lotion for the treatment of scabies. Arch Dermatol. 1990;126:167-70. 2.Franz TJ, Lehman PA, Franz SF, Guin JD. Comparative percutaneous absorption of lindane and permethrin. Arch Dermatol. 1996;132:901-5. Item 7 Answer: (B) Phlebotomy Educational Objective: Choose the appropriate therapy for hepatitis C–related porphyria cutanea tarda. This patient has hepatitis C virus infection, which was probably caused by blood transfusions during one of his many trips to the hospital emergency room when he was a child, and hepatitis C–related porphyria cutanea tarda. Hepatotoxic agents should be avoided. The porphyria should be treated first, followed by treatment of the hepatitis C virus infection to perhaps prevent the development of cirrhosis and possibly hepatocarcinoma. The best treatment for this patient is phlebotomy, to reduce the iron stores. Sodium bicarbonate will alkalinize the urine, but this older therapy is rarely effective for the porphyria. Interferon, with or without ribavirin, is effective for hepatitis C virus infection and may prevent the development of progressive liver disease or cancer of the damaged liver, but it should be used only after the porphyria has been controlled. Bibliography 1.English JC 3rd, Peake MF, Becker LE. Hepatitis C and porphyria cutanea tarda. Cutis. 1996;57:404-8. 2.Herrero C, Lecha M. Management of patients with porphyria cutanea tarda. Photodermatol Photoimmunol Photomed. 1998;14:64-5. 3.Romeo R, Pol S, Berthelot P, Brechot C. Eradication of hepatitis C virus RNA after alpha-interferon therapy. Ann Intern Med. 1994;121:276-7.

Item 8 Answer: (C) Dermatophytosis Educational Objective: Diagnose cutaneous disease based on laboratory results. The potassium hydroxide preparation shows the septate hyphae diagnostic of a dermatophyte infection. Dermatophyte infection of the feet may produce inflammatory and vesicular lesions that may be difficult to distinguish from other infections or inflammatory disorders. Candidiasis produces intertriginous infection and is rarely vesicular. A scraping from a lesion of candidiasis shows abundant budding yeast forms or short hyphae that are not septate. Tinea versicolor is an infection caused by Malassezia furfur that typically presents with hypopigmented or hyperpigmented scaly macules on the trunk and proximal extremities. A potassium hydroxide preparation of scale from a lesion shows short hyphae and yeast forms (so-called spaghetti and meatballs). Herpesvirus infection is unusual on the sole of the foot. A scraping of a herpes vesicle stained with Wright-Giemsa stain would show diagnostic acantholytic ballooned and multinucleated cells. A smear from a lesion of impetigo would contain clumps of gram-positive cocci. Bibliography 1.Masri-Fridling GD. Dermatophytosis of the feet. Dermatol Clin. 1996;14:33-40. 2.Taplin D, Meinking TL. Scabies, lice, and fungal infections. Prim Care. 1989;16:551-76. Item 9 Answer: (E) Pregnancy test Educational Objective: Recognize the teratogenic potential of oral isotretinoin therapy. A woman receiving isotretinoin therapy should have regular laboratory monitoring, including pregnancy testing. At least monthly pregnancy testing is required because of the teratogenic potential of this medication (even if the patient is appropriately following a pregnancy prevention program by using two forms of effective contraception). A complete blood count would not be part of standard monitoring for patients receiving isotretinoin therapy, although very rare hematologic abnormalities have been reported with the use of this drug. Liver function testing is part of the monitoring program; however, abnormalities are not commonly found. One of the major emphases in the development of isotretinoin to differentiate it from oral vitamin A was to reduce the risk of hepatotoxicity. Although triglyceride determinations are important because of the tendency for some patients to develop significant elevations, cholesterol determinations are not routinely required. Isotretinoin may have some effect on the cholesterol level; however, this effect is ephemeral, and it would not have the same short-term consequences as a dramatic elevation in the serum triglyceride level. Urinalysis would not be part of the monitoring of patients receiving isotretinoin therapy, because isotretinoin has negligible risks of renal toxicity. Bibliography 1.Thiboutot DM. An overview of acne and its treatment. Cutis. 1996;57(1 Suppl):8-12. 2.Strauss JS, Rapini RP, Shalita AR, Konecky E, Pochi PE, Comite H, Exner JH. Isotretinoin therapy for acne: results of a multicenter dose-response study. J Am Acad Dermatol. 1984;10:490-6. Item 10 Answer: (E) Her moles increase her chances of developing a melanoma. Educational Objective: Recognize that large numbers of “normal” acquired moles carry an increased risk for melanoma. Large numbers of common acquired moles confer a modest or moderate increase in risk for melanoma. The level of risk depends on the total number of moles present. More than 25 moles by age 20 years confer a modest increase in risk; more than 50 moles confer an even greater risk. The danger is not grave, nor do the moles imply an early onset of melanoma. Congenital nevi in offspring are unrelated to the number of normal acquired moles in parents. Similarly, large numbers of moles in parents do not confer increased risk for melanoma in their children. As the name implies, acquired nevi are not present at birth but develop sometime during late childhood. These nevi tend to be located in sun-exposed areas and are believed to be

secondary to exposure to the ultraviolet radiation in sunshine. The number and anatomic distribution of these moles on the extremities and trunk are related to the frequency and intensity of sun exposure. Like freckles, solar lentigines, and dysplastic nevi, acquired nevi are thought to be caused by clonal expansion of mutated melanocytes in response solarinduced DNA damage. As intermediate end points in the development of cancer, they are markers of patients with a greater tendency to develop melanoma. Acquired nevi function as markers for those patients at increased risk for developing melanoma, but some nevi may actually be precursors of melanoma. Some apparently normal moles change in size, color, or border configuration and prove to be melanoma on biopsy. The question is whether they have always been anaplastic or progressed to that state from a benign precursor mole. In either case, individuals with more than 25 moles by age 20 years should be made aware they are at increased risk for developing a melanoma. Such patients should adhere to a sun-protection program, do skin self-examinations, and report any changing moles or newly developing moles with the irregular features of the ABCD melanoma criteria (asymmetry, border irregularity, color variegation, and diameter > 6 mm). Bibliography 1.Bataille V, Bishop JA, Sasieni P, Swerdlow AJ, Pinney E, Griffiths K, Cuzick J. Risk of cutaneous melanoma in relation to the numbers, types and sites of naevi: a casecontrol study. Br J Cancer. 1996;73:1605-11. 2.Gallagher RP, McLean DI, Yang CP, Coldman AJ, Silver HK, Spinelli JJ, Beagrie M. Suntan, sunburn, and pigmentation factors and the frequency of acquired melanocytic nevi in children. Similarities to melanoma: the Vancouver Mole Study. Arch Dermatol. 1990;126:770-6.

3.Harth Y, Friedman-Birnbaum R, Linn S. Influence of cumulative sun exposure on the prevalence of common acquired nevi. J Am Acad Dermatol. 1992;27:21-4. 4.Harrison SL, Buettner PG, MacLennan R. Body-site distribution of melanocytic nevi in young Australian children. Arch Dermatol. 1999;135:47-52. 5Kelly JW, Rivers JK, MacLennan R, Harrison S, Lewis AE, Tate BJ. Sunlight: a major factor associated with the development of melanocytic nevi in Australian schoolchildren. J Am Acad Dermatol. 1994;30:40-8. Item 11 Answer: (B) Periodic recurrence in the same area Educational Objective: Recognize the various clinical presentations of herpes simplex virus and herpes zoster virus infections. A characteristic feature of herpes simplex virus infection is that, following the primary infection of the skin or mucous membranes, the virus establishes a latent or dormant state within neuronal cells in the ganglia. With reactivation, the virus travels down the nerve fiber to produce recurrent skin infection. Primary infection is more severe than recurrent disease. Vesicles and painful ulcers occur in mucocutaneous areas (mouth and pharynx, cervix and external genitalia); they may be localized but are often widespread, involving the entire mouth or large areas of the genitalia. Accompanying systemic symptoms, such as malaise and fever, are common with primary infection. Recurrent herpes simplex virus infection is usually heralded by prodromal tingling, burning, or dysesthesias, followed by the appearance of clustered vesicles and pustules. Recurrences occur in the same area of the skin. Occasionally, herpes simplex virus may assume a somewhat dermatomal distribution. Herpes zoster is localized disease characterized by unilateral radicular pain and a vesicular eruption that is generally limited to the dermatome innervated by a single spinal or cranial sensory ganglion. It is the result of reactivation of virus that persisted in latent form within ganglia following earlier infection with varicella. Prodromal pain and paresthesia in the involved dermatome usually precede the eruption by several days. A distinctive feature of herpes zoster is the localization and distribution of the eruption, which is almost always unilateral, does not cross the midline, and is generally limited to the area of skin innervated by a single sensory ganglion. Widespread disease may occur in immunocompromised patients. Both herpes simplex virus and herpes zoster virus infections are characterized by the presence of multiple vesicles. Bibliography 1.Cohen JI, Brunell PA, Straus SE, Krause PR. Recent advances in varicella-zoster virus infection. Ann Intern Med. 1999; 130:922-32.

2.McCrary ML, Severson J, Tyring SK. Varicella zoster virus. J Am Acad Dermatol. 1999;41:114. 3.Pereira FA. Herpes simplex: evolving concepts. J Am Acad Dermatol. 1996;35:503-20. Item 12 Answer: (C) Multiple childhood sunburns Educational Objective: Understand the importance of childhood sunburns in the pathogenesis of melanoma. The development of skin cancer is most strongly correlated with exposure to the ultraviolet radiation in sunlight. Exposure to fluorescent light has never been proved to cause the development of melanoma. There is a prolonged lag period between the actual exposure to ultraviolet radiation and the development of melanoma. The types of exposure listed in the options, other than the multiple childhood sunburns, would be insufficient on their own to cause skin cancer. Several aspects of the relationship between melanoma and ultraviolet radiation have been puzzling. Why do some patients develop a melanoma, whereas others develop a basal cell or squamous cell carcinoma? Why do the tumors occur on the trunk in some patients and on the head and neck of other patients? Why do skin cancers appear in the third or fourth decade in some patients and not until the seventh or eighth in other patients? A paradigm has been developed to help answer many of these questions. It appears that several variables play a role: (A) The genetic make-up of patients in terms of their response to ultraviolet radiation: complexion, ability to tan, tendency to burn. (B) The timing of the exposure to ultraviolet radiation: childhood or adulthood. (C) The intensity of the exposure: strong enough to result in a sunburn (redness, erythema) or less intense (suberythemal). (D) Frequency of the ultraviolet radiation exposure: chronic, such as that in outdoor workers, or intermittent, such as that occurring during weekend recreation or once-a-year vacations in sunny climates. (E) The skin areas exposed: the usually exposed skin of the face, neck, and lower arms or the intermittently exposed skin of the chest, back, and shoulders. (F) The two predominant cells in the epidermis exposed to ultraviolet radiation: keratinocytes and melanocytes. Actinic keratoses and squamous cell carcinoma are derived from the actively replicating keratinocytes that reside in the basal layer of the epidermis, just above the basement membrane separating the epidermis from the dermis. These tumors are believed to be caused by chronic exposure to ultraviolet radiation of skin that is usually exposed to the sun, including the head, neck, and upper extremities. The tumors appear late in life (sixth, seventh, and eighth decades). Ultraviolet radiation damages the DNA of the keratinocytes, resulting in characteristic thymine dimers. If the cells are heavily damaged, p53 tumor suppresser genes (and most likely, other tumor genes) begin apoptotic sequences and remove the irreparable cell. However, if damage is not so severe, p53 tumor suppresser genes slow the cell cycle until excision repair of the faulty DNA strands can be accomplished. The normal cell then re-enters the proliferative pool or differentiates toward keratin production, moves up the epidermis, and dies as it forms part of the stratum corneum. However, with continued chronic exposure to ultraviolet radiation, some mutations in the DNA, especially in the p53 tumor suppresser gene itself, occur and accumulate. Ultimately, with clonal expansion, a squamous cell in situ (actinic keratosis) is formed, which may or may not progress to an invasive squamous cell carcinoma. Melanomas are derived from the much less numerous melanocytes, which also reside in the basal layer of the epidermis. They provide the melanin pigment that gives persons their complexion and the ability to tan in response to sun exposure. These cells normally do not divide, and protecting them from destruction is a priority. Consequently, with heavy ultraviolet radiation exposure, considerable DNA damage is done.

Bibliography 1.Gilchrest BA, Eller MS, Geller AC, Yaar M. The pathogenesis of melanoma induced by ultraviolet radiation. N Engl J Med. 1999;340:1341-8.

Item 13 Answer: (B) Skin biopsy for routine processing Educational Objective: Order an appropriate test to evaluate a patient with central facial erythema. In patients with central facial erythema, it may be difficult to distinguish relatively benign cutaneous disorders, such as rosacea, from diseases with potential systemic consequences, such as systemic lupus erythematosus (SLE) and dermatomyositis. The characteristic central facial papules and pustules usually allow for a clinical diagnosis of rosacea, and a skin biopsy is not necessary. However, if SLE or dermatomyositis cannot be distinguished from rosacea on dermatologic examination, a routine skin biopsy with subsequent histologic examination by a competent dermatopathologist can reveal interface change, which is highly suggestive of either SLE or dermatomyositis. These conditions could then be evaluated more fully as appropriate. Histologic interface change would not be found in patients with rosacea. Direct immunofluorescence microscopy is less useful, because false-positive results can occur in patients with rosacea, and this test can be positive in patients with SLE or dermatomyositis. An antinuclear antibody test can be positive in patients with SLE or dermatomyositis; however, as many as 25% to 30% patients older than 65 years have positive antinuclear antibody determinations. A complete blood count could reveal hematologic abnormalities in patients with SLE, but this would not be a specific finding. The serum creatine kinase level could be elevated in patients with dermatomyositis, but it would not be helpful in distinguishing SLE from rosacea. Bibliography 1.Helm KF, Menz J, Gibson LE, Dicken CH. A clinical and histopathologic study of granulomatous rosacea. J Am Acad Dermatol. 1991;25(Pt 1):1038-43. 2.Lerner EA, Lerner MR. Whither the ANA? Arch Dermatol. 1987;123:358-62. 3.Nunzi E, Rebora A, Hamerlinck F, Cormane RH. Immunopathological studies on rosacea. Br J Dermatol. 1980;103:543-51. Item 14 Answer: (C) Push on an intact blister to see if it can be extended. Educational Objective: Correlate the clinical findings of autoimmune blistering diseases with the level of pathologic process in the skin. This 68-year-old man has bullous pemphigoid, which is a subepidermal blistering disorder. The response to the bedside diagnostic maneuver of pressing on the intact blister edge allows discrimination between an intraepidermal blistering process and a subepidermal process. In this patient with bullous pemphigoid, it would be expected that the blister would not advance with application of pressure, because its pathologic location is subepidermal. Although a skin biopsy is necessary, the results of this test take days. A Tzanck test is helpful for identifying giant cells in herpes viral infections, and rubbing normal skin or culturing for atypical mycobacteria has no diagnostic significance. Bibliography 1.Scott JE, Ahmed AR. The blistering diseases. Med Clin North Am. 1998;82:1239-83. 2.Korman NJ. Bullous pemphigoid. The latest in diagnosis, prognosis, and therapy. Arch Dermatol. 1998;134:1137-41. Item 15 Answer: (B) Psoriasis Educational Objective: Recognize and distinguish arthritis that may be associated with cutaneous disease. Many common forms of arthritis are associated with skin disease. Recognition of the cutaneous findings of arthritis may be helpful in establishing the diagnosis. This patient has characteristic nail and joint findings of psoriatic arthritis. Onset of arthritis is usually insidious, and most patients manifest a peripheral asymmetric oligoarthritis that involves the small joints of the hands and feet, the large joints of the legs, or a combination of both. Onychodystrophy, onycholysis, nail pitting, and subungual keratosis are more strongly associated with arthritis than with psoriasis alone. Hyperuricemia may occur in patients with psoriasis. Gouty tophi

appear late in the course of gout. They occur in the olecranon bursa, infrapatella, Achilles tendon, subcutaneous tissues on the extensor surface of the forearm, and overlying joints and occasionally around the helix of the ear. Nail changes are not seen in gout. Nail disease caused by dermatophyte infection (onychomycosis) may resemble psoriatic nail changes, with subungual hyperkeratosis, thickening, and onycholysis; however, there is no joint disease in this condition. The diagnosis of onychomycosis can be confirmed with nail clippings that are examined with potassium hydroxide or with fungal cultures. Rheumatoid arthritis may be associated with rheumatoid nodules in 25% to 50% of patients; they tend to occur when the disease is most active. Rheumatoid vasculitis and pyoderma gangrenosum also may be seen in association with rheumatoid arthritis. Nail changes typically do not occur. The joints most commonly affected by sarcoidosis are the knees, ankles, elbows, wrists, and small joints of the hands. Affected joints are swollen, warm, tender, and painful. Skin lesions include lupus pernio (violaceous indurated lesions with a predilection for the nose, ears, lips, and face), skin plaques, papules, subcutaneous nodules, and erythema nodosum. Nail changes are not seen. Bibliography 1.Gladman DD. Psoriatic arthritis. Rheum Dis Clin North Am. 1998;24:829-44. 2.McGonagle D, Conaghan PG, Emery P. Psoriatic arthritis: a unified concept twenty years on. Arthritis Rheum. 1999; 42:1080-6. 3.Espinoza LR, van Solingen R, Cuellar ML, Angulo J. Insights into the pathogenesis of psoriasis and psoriatic arthritis. Am J Med Sci. 1998;316:271-6. 4.Ross EL, D’Cruz D, Morrow WJ. Pathogenic mechanisms in psoriatic arthritis. Hosp Med. 1998;59:534-8. Item 16 Answer: (E) Hot beverages Educational Objective: Recognize that ingesting hot liquids is an important cause of flushing in patients with rosacea. Flushing can be a particularly disabling manifestation of rosacea. All of the stimuli mentioned, including alcohol, caffeine, spicy foods, sun exposure, and hot beverages, have been implicated in making flushing worse. Studies have shown that overheating of the oral cavity, such as with hot liquids, perhaps through stimulation of the carotid sinus, can reliably produce flushing. Exposure to caffeinated beverages and to other stimuli did not readily produce flushing if temperature was controlled. This is relevant from the therapeutic standpoint, because rapid cooling of the oral cavity, such as with ice chips, can abort the flushing that follows drinking hot beverages. Bibliography 1.Wilkin JK. Oral thermal-induced flushing in erythematotelangiectatic rosacea. J Invest Dermatol. 1981;76:15-8. 2.Wilkin JK. Effect of nadolol on flushing reactions in rosacea. J Am Acad Dermatol. 1989;20(Pt 1):202-5. Item 17 Answer: (B) Drug-induced linear IgA bullous dermatosis Educational Objective: Recognize the causes of drug-induced autoimmune blistering diseases. This 28-year-old woman has developed cellulitis as a complication of her atopic dermatitis. The initiation of vancomycin therapy followed by the onset of a blistering disorder strongly suggests the diagnosis of linear IgA bullous dermatosis. Bullous impetigo is highly unlikely, because the patient was treated with vancomycin. Acute contact dermatitis from the hospital bed sheets would be a highly unlikely cause of a localized blistering reaction when large skin surface areas are exposed. Atopic dermatitis is not exacerbated by vancomycin. Pemphigus is an intraepidermal blistering process, and thus, it would be expected that the patient’s blisters could be advanced with application of pressure. Bibliography 1.Baden LA, Apovian C, Imber MJ, Dover JS. Vancomycin-induced linear IgA bullous dermatosis. Arch Dermatol. 1988;124:1186-8. 2.Carpenter S, Berg D, Sidhu-Malik N, Hall RP 3d, Rico MJ. Vancomycin-associated linear IgA

dermatosis. A report of three cases. J Am Acad Dermatol. 1992;26:45-8. 3Nousari HC, Kimyai-Asadi A, Caeiro JP, Anhalt GJ. Clinical, demographic, and immunohistologic features of vancomycin-induced linear IgA bullous disease of the skin. Report of 2 cases and review of the literature. Medicine (Baltimore). 1999;78:1-8. Item 18 Answer: (C) Resolve Educational Objective: Understand that the level of immunosuppression correlates with HIVassociated skin disease and that highly active antiretroviral therapy (HAART) may reverse these manifestations. Skin lesions in patients with HIV infection correlate with the level of immunosuppression, and effective therapy can result in a “spontaneous” healing of the lesions. In this case, the patient presented with molluscum contagiosum that was associated with a high level of immunosuppression. Fortunately, with the newer forms of therapy, viral loads and the level of immune dysfunction may be reversed, and uncontrollable cutaneous disorders, including viral infections, may resolve or may be controlled more easily. Bibliography 1.Goldstein B, Berman B, Sukenik E, Frankel SJ. Correlation of skin disorders with CD4 lymphocyte counts in patients with HIV/AIDS. J Am Acad Dermatol. 1997;36:262-4.

2.Spach DH, Colven R. Resolution of recalcitrant hand warts in an HIV-infected patient treated with potent antiretroviral therapy. J Am Acad Dermatol. 1999;40:818-21. 3.Rich JD, Mylonakis E, Nossa R, Chapnick RM. Highly active antiretroviral therapy leading to resolution of porphyria cutanea tarda in a patient with AIDS and hepatitis C. Dig Dis Sci. 1999;44:1034-7. 4.Hicks CB, Myers SA, Giner J. Resolution of intractable molluscum contagiosum in a human immunodeficiency virus–infected patient after institution of antiretroviral therapy with ritonavir. Clin Infect Dis. 1997;24:1023-5. Item 19 Answer: (D) Paraneoplastic pemphigus Educational Objective: Recognize the clinical presentation, histopathologic characteristics, and immunofluorescence findings of paraneoplastic pemphigus. Paraneoplastic pemphigus is a recently described entity that occurs most often in patients with a history of lymphoma or who currently have lymphoma. Some patients have been described in whom there is no evidence of malignancy, but in such patients, it is reasonable to search for a possible neoplasm. This man’s lesions are characteristic of pemphigus, and the presence of a positive immunofluorescence study on rat bladder is highly suggestive of the paraneoplastic variant. Impetigo may manifest similar skin lesions but not intraoral lesions. The histopathologic and immunopathologic findings are not those of cutaneous lupus erythematosus, which would include an interface dermatitis and granular deposition of immunoglobulin along the dermal-epidermal junction. Sweet’s syndrome is manifested by cellulitic-appearing plaques. The glucagonoma syndrome might be associated with weight loss and an angular cheilitis, but the skin lesions are an intertriginous erosive dermatitis. Bibliography 1.Udey MC, Stanley JR. Pemphigus—diseases of antidesmosomal autoimmunity. JAMA. 1999;282:572-6. 2.Robinson ND, Hashimoto T, Amagai M, Chan LS. The new pemphigus variants. J Am Acad Dermatol. 1999;40:649-71. 3.Nousari HC, Deterding R, Wojtczack H, Aho S, Uitto J, Hashimoto T, Anhalt GJ. The mechanism of respiratory failure in paraneoplastic pemphigus. N Engl J Med. 1999;340:1406-10.

Item 20 Answer: (A) Tetracycline and niacinamide Educational Objective: Treat patients with autoimmune blistering disorders. This 80-year-old man has characteristic clinical and histopathologic findings of bullous pemphigoid. Because of the relatively benign presentation of his illness and his numerous systemic problems, including insulin-dependent diabetes mellitus and hypertension, the initiation of tetracycline and niacinamide therapy should be considered first before using prednisone or prednisone plus azathioprine. The combination of tetracycline and niacinamide alone has proved effective in the treatment of some patients with bullous pemphigoid. Methotrexate is not particularly effective in the treatment of bullous pemphigoid, and dicloxacillin has no effect on this disorder. Bibliography 1.Berk MA, Lorincz AL. The treatment of bullous pemphigoid with tetracycline and niacinamide.

A preliminary report. Arch Dermatol. 1986;122:670-4. 2.Hornschuh B, Hamm H, Wever S, Hashimoto T, Schroder U, Brocker EB, Zillikens D. Treatment of 16 patients with bullous pemphigoid with oral tetracycline and niacinamide and topical clobetasol. J Am Acad Dermatol. 1997;36:101-3. Item 21 Answer: (B) Primary systemic amyloidosis Educational Objective: Recognize the cutaneous features of primary systemic amyloidosis. This patient has primary systemic amyloidosis, manifested by “pinch” purpura. This condition is associated with the widespread deposition of amyloid in multiple tissues, including the skin, as demonstrated by the findings on skin biopsy. Primary systemic amyloidosis is associated with multiple myeloma in almost all instances, and this patient demonstrated a monoclonal paraprotein on serum protein electrophoresis. Actinic purpura is manifested by ecchymoses on the arms and is caused by capillary fragility. Idiopathic thrombocytopenic purpura presents as petechiae, not macular purpura. Small-vessel vasculitis is manifested as palpable purpura, most often on the legs. Kaposi’s sarcoma may occur in HIV infection as a macular, purpuric lesion but would not have occurred following straining during defecation. Bibliography 1.Kyle RA, Gertz MA. Primary systemic amyloidosis: clinical and laboratory features in 474 cases. Semin Hematol. 1995;32:45-59. 2.Kyle RA. Sequence of testing for monoclonal gammopathies. Arch Pathol Lab Med. 1999;123:114-8. 3.Daoud MS, Lust JA, Kyle RA, Pittelkow MR. Monoclonal gammopathies and associated skin disorders. J Am Acad Dermatol. 1999;40:507-35. Item 22 Answer: (D) Discontinue trimethoprim-sulfamethoxazole, and hospitalize the patient for intravenous immunoglobulin therapy. Educational Objective: Treat a patient with toxic epidermal necrolysis. This young woman has the early signs and symptoms of toxic epidermal necrolysis secondary to her antibiotic therapy. The approach to treatment is to discontinue the medications and begin intravenous immunoglobulin therapy, which blocks the apoptotic activity of the Fas ligand on keratinocytes and inhibits epidermal necrosis. The use of prednisone in such patients continues to be controversial and is often associated with increased morbidity. Bibliography 1.Viard I, Wehrli P, Bullani R, Schneider P, Holler N, Salomon D, et al. Inhibition of toxic epidermal necrolysis by blockade of CD95 with human intravenous immunoglobulin. Science. 1998;282:490-3. 2.Sanwo M, Nwadiuko R, Beall G. Use of intravenous immunoglobulin in the treatment of severe cutaneous drug reactions in patients with AIDS. J Allergy Clin Immunol. 1996; 98(Pt 1):1112-5.

Item 23 Answer: (D) Triamcinolone acetonide Educational Objective: Select the best initial therapy for eczema of the hands. The mainstay of treatment for eczematous eruptions is the use of topical glucocorticoids. As with most therapeutic medications, an agent must be present in an effective concentration at the site of the pathologic condition. Therefore, topical glucocorticoids are not useful for subcutaneous diseases, such as erythema nodosum, or for deeper dermal inflammatory conditions, such as urticaria. They are ideally suited for superficial inflammatory conditions, such as eczematous eruptions, in which the histopathologic changes are confined to the upper dermis and epidermis. It is also important to exclude inflammatory conditions that produce secondary epidermal inflammation but that would be made worse by topical glucocorticoids, such as infections with dermatophytes or Candida. Topical calcipotriene is a relatively new treatment that has proved to be effective in the management of patients with psoriasis. It is generally combined with topical glucocorticoid therapy to achieve both a rapid response from the glucocorticoid and a more sustained benefit from the calcipotriene. Topical retinoids (such as tretinoin or the newer tazarotene) have been used in the treatment of comedonal acne but also have been used more recently in the treatment of psoriasis, playing much the same role as calcipotriene. Neosporin should be avoided because of contact dermatitis, which is a particular risk on skin that is already eczematized. Patients with eczematized skin often become allergic to the Neosporin and may develop a contact dermatitis. This causes diagnostic and therapeutic confusion in a patient with a pre-existing eczematous condition. The combination product clotrimazole–betamethasone dipropionate should be avoided because of the potency of the glucocorticoid and the absence of fungal infection requiring clotrimazole. Bibliography 1.Ruzicka T. Atopic eczema between rationality and irrationality. Arch Dermatol. 1998;134:1462-9. 2.Feldman SR, Clark AR. Psoriasis. Med Clin North Am. 1998;82:1135-44. Item 24 Answer: (C) History and physical examination Educational Objective: Understand the role of laboratory studies in the management of melanoma. The most critical issue for patient survival once the diagnosis of melanoma is established is whether metastases have occurred before the primary cutaneous lesion was successfully excised. In approximately 20% of clinical stage I and II disease (local disease), patients are harboring undetectable micrometastases in the lymph nodes or in distant organs. To date, the Breslow thickness has been the best prognostic indicator for survival, with the risk of micrometastases correlated with increasing thickness. In prospective studies, elective lymph node dissection was shown to be costly, associated with considerable morbidity, and ineffective in improving survival. Lymphatic mapping and selective or sentinel node biopsy are being investigated as a less morbid and less expensive way to classify patients with melanoma. It is hoped that when micrometastases are found in the sentinel node, subsequent full lymph node dissection and adjuvant therapy when the tumor burden is low will improve survival. Similarly, it is hoped that when the sentinel node is negative for tumor, patients will remain tumor-free. Both hypotheses have not yet been proved. In the meantime, it has become evident that neither the inexpensive tests, such as the serum lactate dehydrogenase level and chest radiography, nor the more costly studies, such as CT, MRI, and positron emission tomography, have the ability to identify small foci of tumors cells. Consequently, the main purpose of frequent follow-up visits is to check for local recurrences, check for new lesions, and perform a complete history and physical examination. Laboratory studies should be ordered as a function of the clinical symptoms and findings. With frequent follow-up visits, clinically detected metastases should be found as early as possible, and appropriate therapy should be rendered when it is most effective. One promising blood test is the tumor-associated antigen TA-90 immune complex assay. It may be useful for staging and for postoperative monitoring for tumor recurrence. Preliminary data indicate that elevated preoperative levels of TA-90 immune complex are associated with decreased survival and that persistent elevated levels postoperatively are similarly ominous.

Bibliography 1.Huang CL, Provost N, Marghoob AA, Kopf AW, Levin L, Bart RS. Laboratory tests and imaging studies in patients with cutaneous malignant melanoma. J Am Acad Dermatol. 1998;39:451-63. 2.Weiss M, Loprinzi CL, Creagan ET, Dalton RJ, Novotny P, O’Fallon JR. Utility of follow-up tests for detecting recurrent disease in patients with malignant melanomas. JAMA. 1995;274:1703-5. 3.Terhune MH, Swanson N, Johnson TM. Use of chest radiography in the initial evaluation of patients with localized melanoma. Arch Dermatol. 1998;134:569-72. 4.Kelley MC, Jones RC, Gupta RK, Yee R, Stern S, Wanek L, Morton DL. Tumor-associated antigen TA-90 immune complex assay predicts subclinical metastasis and survival for patients with early stage melanoma. Cancer. 1998;83:1355-61. Item 25 Answer: (D) HIV infection Educational Objective: Recognize persons at risk for severe infestation with scabies. Immunocompromised patients (particularly those with AIDS) and institutionalized patients may develop widespread scabies with extensive scaling that may not itch. This uncommon variant, crusted scabies, also known as Norwegian scabies, is often misdiagnosed as eczema, psoriasis, or ichthyosis. It is associated with a vast number of mites and is highly contagious. Crusted scabies does not occur with any frequency in patients with the other diseases listed. Cutaneous lesions associated with acute myelogenous leukemia include leukemic infiltrates in the skin, plaques of Sweet’s syndrome, and less specific petechiae and purpura. Patients with cystic fibrosis have increased amounts of electrolytes in the sweat, which leads to excessive skin wrinkling when the palms and soles are immersed in water. The only definitive mucocutaneous lesion attributed to Epstein-Barr virus infection is oral hairy leukoplakia. Patients with Epstein-Barr virus infection who receive antibiotic therapy, particularly ampicillin, are prone to develop a widespread pruritic morbilliform eruption. There is no increased incidence of scabies in patients with psoriasis. Bibliography 1.Orkin M. Scabies in AIDS. Semin Dermatol. 1993;12:9-14. Item 26 Answer: (D) Rosacea Educational Objective: Recognize the clinical appearance of rosacea, and distinguish it from other inflammatory facial lesions. This patient’s history and clinical appearance are characteristic of rosacea, a chronic dermatosis of middle-aged and older patients. Rosacea begins insidiously with transient erythema, followed by telangiectasias, papules, pustules, and, rarely, nodules. Rhinophyma, a late sequela, is more common in men. Patients often give a history of facial flushing following ingestion of hot liquids or alcohol. Dermatophytosis involving the face (tinea faciei) is an uncommon but often missed fungal infection. Lesions are asymmetric, erythematous, and sometimes annular. A potassium hydroxide examination of surface scale shows diagnostic hyphae. Allergic contact dermatitis is manifested as a pruritic papular and vesicular eruption, occurring at the site of antigen exposure. It typically begins abruptly and clears in a few weeks. Discrete papules and pustules are not typical of allergic contact dermatitis. An irritant contact dermatitis is produced by repetitive exposure to a harsh chemical and typically results in confluent erythema with lichenification, scale, and fissuring. Facial eruptions associated with systemic lupus erythematosus may be difficult to differentiate from rosacea. Symmetric malar macular erythema is seen in systemic lupus erythematosus; sharply demarcated erythematous, hyperpigmented and hypopigmented plaques on the scalp and sun-exposed areas are typical of cutaneous discoid lupus erythematosus lesions. Discrete small papules and pustules typical of rosacea are not seen in systemic lupus erythematosus. Photosensitivity, systemic symptoms, and a positive antinuclear antibody titer are other manifestations of systemic lupus erythematosus. Seborrheic dermatitis is a chronic cutaneous disorder that appears as welldemarcated, confluent, macular erythema with greasy scale. Lesions are most prominent on the scalp, eyebrows, beard area, nasolabial creases, and central chest.

Bibliography 1.Higgins E, du Vivier A. Alcohol intake and other skin disorders. Clin Dermatol. 1999;17:43741. 2.Wilkin JK. Use of topical products for maintaining remission in rosacea. Arch Dermatol. 1999;135:79-80. Item 27 Answer: (A) Teach her how to do monthly skin self-examinations. Educational Objective: Manage a patient at high risk for developing melanoma. This patient’s melanoma was relatively thin and had no other characteristics that would increase the likelihood of metastases (for example, ulceration or regression). The risk that she is harboring clinically undetectable micrometastases is low, and there is a greater than 95% chance that she will be cured. The importance of frequent follow-up examinations is to detect any local, regional, or distant recurrences as early as possible, when prompt intervention would be most beneficial. In addition, this patient has a relative risk of 8.5 of developing another melanoma. About 75% of new melanomas occur in a different anatomic region than the initial melanoma. Frequent surveillance allows very early detection of new melanomas, when they are readily curable. Skin self-examination is another strategy for early detection of new melanomas. Patients can be taught the essentials of doing total skin examinations using a combination of wall-mounted and hand-held mirrors. Patients should be instructed to report any new moles, any changing moles, or any lesions that have any of the ABCD changes associated with melanoma (asymmetry, border irregularity, color variegation, and diameter > 6 mm). Monthly skin selfexamination has been shown to be an effective method of early tumor detection. In addition, it focuses patients’ attention on their propensity to form skin tumors and heightens their compliance with a sun-protection program. Laboratory studies and even sophisticated diagnostic modalities such as MRI and CT have not been effective in detecting micrometastases. Sentinel node biopsy is not recommended for early, thin melanoma. Sentinel node biopsy is an interventional staging technique that would have an extremely low benefit in tumors less than 1 mm in thickness. Similarly, adjuvant therapy with interferon is not warranted in this low-risk patient. Although this patient would benefit from education on appropriate sun-avoidance techniques, formal psychologic counseling is not required. Bibliography 1.Berwick M, Begg CB, Fine JA, Roush GC, Barnhill RL. Screening for cutaneous melanoma by skin self-examination. J Natl Cancer Inst. 1996;88:17-23. 2.Miller DR, Geller AC, Wyatt SW, Halpern A, Howell JB, Cockerell C, et al. Melanoma awareness and self-examination practices: results of a United States survey. J Am Acad Dermatol. 1996;34:962-70. 3.Johnson TM, Hamilton T, Lowe L. Multiple primary melanomas. J Am Acad Dermatol. 1998;39:422-7. 4.Marghoob AA, Slade J, Kopf AW, Salopek TG, Rigel DS, Bart RS. Risk of developing multiple primary cutaneous melanomas in patients with the classic atypical-mole syndrome: a case-control study. Br J Dermatol. 1996;135:704-11. Item 28 Answer: (E) A fixed-drug eruption Educational Objective: Recognize the clinical findings of a fixed-drug eruption Although the anatomic area of the eruption suggests a recurrent herpes simplex reaction, this patient has the characteristic history and physical findings of a fixed-drug eruption. Fixed-drug eruption may represent a localized variant of erythema multiforme. Inflammatory lesions with residual hyperpigmentation recur at the same site with each exposure to the offending drug.

The pathologic mechanism of a fixed-drug eruption is poorly understood, but the pathologic process occurs at the junction of the epidermis and dermis, often resulting in the deposition of epidermal melanin in the upper dermis, which causes a brown-gray color at the site of the reaction. The area is too localized for a contact dermatitis from laundry detergent, and a recurrent herpes simplex infection would be expected to present as grouped vesicles. It would

be very unusual for recurrent urticaria to occur at the same site, and persisting dyspigmentation would not be expected. A fungal infection should present as a scaling macule and usually would not resolve without treatment. Bibliography 1.Wolverton SE. Update on cutaneous drug reactions. Adv Dermatol. 1997;13:65-84. 2.Vervloet D, Durham S. Adverse reactions to drugs. BMJ. 1998;316:1511-4. Item 29 Answer: (A) Topical clobetasol Educational Objective: Recognize that systemic glucocorticoids are relatively contraindicated in patients with psoriasis. The development of generalized pustular psoriasis is one of the most impressive complications of dermatologic illness. Although this condition can occur de novo, it often follows the discontinuation of a systemic glucocorticoid. For this reason, most dermatologists believe that the use of systemic glucocorticoids in patients with psoriasis is relatively contraindicated. Therefore, topical clobetasol, a superpotent glucocorticoid, is the therapy of choice for this patient with localized disease. For this reason, patients who require systemic glucocorticoids for other conditions, such as systemic lupus erythematosus, autoimmune bullous diseases (for example, bullous pemphigoid), and psoriasis, are particularly difficult to manage. Oral prednisone in a 2-week taper is the preferred method of treating generalized rhus dermatitis in patients who do not have psoriasis. The methylprednisolone dose pack, although sometimes effective, is associated with rebound owing to the short duration of therapy. Oral prednisone tapers produce a smoother taper than intramuscular dosing with depocorticosteroids. This is because of less predictable absorption with the intramuscular route of administration. Nonsedating antihistamines such as loratadine have no role in the treatment of rhus dermatitis because the pruritus is not primarily histamine-mediated. Bibliography 1.Feldman SR, Clark AR. Psoriasis. Med Clin North Am. 1998;82:1135-44. 2.Baker H. Corticosteroids and pustular psoriasis. Br J Dermatol. 1976;94(Suppl 12):83-8. Item 30 Answer: (A) Refer him to a dermatologist for a biopsy of the lesion. Educational Objective: Recognize lentigo maligna, and understand that it is a precursor of melanoma. This patient has a lentigo maligna, which is an in situ stage of melanoma. Although completely flat, it is asymmetric, the border is notched, and there is variegation of color, all of which are characteristic warning signs of melanoma. The most appropriate option in managing this patient is to perform a biopsy of the lesion for histologic confirmation of the diagnosis. Removing a pigmented lesion with liquid nitrogen or a laser without a diagnosis is inappropriate. There are no laboratory tests other than biopsy that would be of any help. Lentigo maligna usually develops in elderly patients in the seventh or eighth decade who have other signs of chronic sun exposure. It typically expands slowly and develops as a pigmented lesion in a sun-exposed location (for example, the head and neck). Less than 20% of lentigo maligna develop on sites other than the head and neck (usually on the extremities of women and on the trunk of men). The lesion often remains macular (flat) for many years before it becomes invasive and palpable tumor is evident. The lag time before invasion occurs is unpredictable, but it is usually several years. The accepted range of progression from lentigo maligna to lentigo maligna melanoma is from 15% to 30%. Some authorities believe that given enough time, most lentigo maligna would progress to lentigo maligna melanoma, but because of the incidence in the elderly population, many patients die before invasion occurs. Once invasion occurs, the prognosis of lentigo maligna melanoma depends on the thickness of the tumor and is similar to that of comparable superficial spreading and nodular melanomas of the same thickness. There has been a lingering misconception that lentigo maligna melanoma has a more benign clinical course. The clinical presentation of lentigo maligna has all the signs associated with melanoma, including asymmetry; irregular, blurred, or notched borders; large size; and often dramatic variegation of color. Colors range from shades of tan and brown to red and blue. The size may

vary from less than 1 cm to several centimeters and may cover large areas of the cheek, forehead, or scalp. If palpable areas are present, a presumptive diagnosis of lentigo maligna melanoma should be made. The treatment of lentigo maligna is surgical excision. The entire lesion should be removed to prevent recurrence and to analyze the entire specimen for foci of invasion. The latter would change the prognosis dramatically. Lentigo maligna has no associated mortality rate, whereas the survival of patients with lentigo maligna melanoma depends on tumor thickness and level of invasion. Mohs’ micrographic surgery is the procedure used to treat lentigo maligna melanoma, with cure rates approaching 97%. Routine surgical excision, cryosurgery, and radiation therapy are associated with recurrence rates of about 7% to 10%. Radiation therapy is an alternative to surgery and may be a reasonable choice in infirm elderly patients who are not able to undergo surgery. For some large, flat lesions, radiation therapy may be better than extensive excision, which may result in cosmetic or functional deformity. Solar lentigo and seborrheic keratosis figure prominently in the differential diagnosis of lentigo maligna. Solar lentigines (liver spots or sun spots) are usually small (< 1 cm), evenly pigmented, tan to brown lesions with a regular border. They occur most often on sundamaged skin of the back of the hands, temple, forehead, and scalp. Larger, darker lesions may cause some confusion with lentigo maligna. Seborrheic keratoses are also light tan to dark brown but usually are palpable and have a scaly surface and sharply demarcated borders. Lentigo maligna is not a marker of any internal or systemic diseases. Bibliography 1.Cohen LM, McCall MW, Zax RH. Mohs micrographic surgery for lentigo maligna and lentigo maligna melanoma. A follow-up study. Dermatol Surg. 1998;24:673-7. 2.Geara FB, Ang KK. Radiation therapy for malignant melanoma. Surg Clin North Am. 1996;76:1383-98. 3.Cox NH, Aitchison TC, MacKie RM. Extrafacial lentigo maligna melanoma: analysis of 71 cases and comparison with lentigo maligna melanoma of the head and neck. Br J Dermatol. 1998;139:439-43. Item 31 Answer: (B) Lithium Educational Objective: Recognize medications that may exacerbate psoriasis. Of the drugs listed, lithium is the only one associated with an exacerbation of psoriasis. Psoriasis is a common disease; patients with psoriasis often require systemic medications for concomitant disorders. It is important to realize that many drugs may aggravate existing disease or even precipitate progression to pustular psoriasis. The drugs most frequently implicated in the exacerbation of psoriasis include beta-receptors (particularly propranolol), lithium, synthetic antimalarial agents (quinacrine hydrochloride and chloroquine), nonsteroidal anti-inflammatory drugs (phenylbutazone, indomethacin, and ibuprofen), angiotensin-converting enzyme inhibitors (captopril), and interferon-alfa. None of these medications is absolutely contraindicated in patients with psoriasis, but patients should be warned that their skin disease may worsen. The other medications that this patient is taking generally do not worsen psoriasis. Bibliography 1.Tsankov N, Kazandjieva J, Drenovska K. Drugs in exacerbation and provocation of psoriasis. Clin Dermatol. 1998;16:333-51. 2.Abel EA, DiCicco LM, Orenberg EK, Fraki JE, Farber EM. Drugs in exacerbation of psoriasis. J Am Acad Dermatol. 1986;15(Pt 1):1007-22.

Item 32 Answer: (D) Pelvic ultrasonography Educational Objective: Evaluate a 45-year-old woman with dermatomyositis. This patient presents with the classic features of dermatomyositis. Malignancy is common in patients with dermatomyositis but occurs more frequently in older patients. Although this woman is 45 years old, she should have an evaluation for a possible malignancy. A likely site for a cancer in such a patient would be the pelvis. Ovarian cancer appears to be overrepresented in most recent case studies; therefore, the test most likely to detect a malignancy in this patient is pelvic ultrasonography. Barium enema would be a correct answer if the patient were older than 60 years of age. Although it could possibly detect metastatic disease, a brain scan would not be useful in this patient. Thyroid cancer is not overrepresented in these patients, and thus, a thyroid scan would not be useful. Carcinoembryonic antigen (CEA) is a nonspecific test and is not useful for cancer screening in this patient. Bibliography 1.Davis MD, Ahmed I. Ovarian malignancy in patients with dermatomyositis and polymyositis: a retrospective analysis of fourteen cases. J Am Acad Dermatol. 1997;37:730-3. 2.Sigurgeirsson B, Lindelof B, Edhag O, Allander E. Risk of cancer in patients with dermatomyositis or polymyositis. A population-based study. N Engl J Med. 1992;326:363-7.

3.Marie I, Hatron PY, Levesque H, Hachulla E, Hellot MF, Michon-Pasturel U, et al. Influence of age on characteristics of polymyositis and dermatomyositis in adults. Medicine (Baltimore). 1999;78:139-47. Item 33 Answer: (B) Discontinue phenytoin, and continue ceftazidime. Educational Objective: Identify the early signs of phenytoin hypersensitivity reaction. This patient is developing the early signs of phenytoin hypersensitivity reaction, which often presents as fever, a generalized erythematous macular or papular eruption, periorbital or facial edema, lymphadenopathy, abnormal liver function tests, and leukocytosis with eosinophilia. Because there is still a question about a possible infection, it is best to discontinue the phenytoin but maintain antibiotic coverage until the results of the cultures are known. In addition, the possibility of infection would preclude the immediate use of intravenous methylprednisolone sodium succinate. Carbamazepine can cross-react with phenytoin; therefore, it is an inappropriate antiseizure agent in this patient. Bibliography 1.Shear NH, Spielberg SP. Anticonvulsant hypersensitivity syndrome. In vitro assessment of risk. J Clin Invest. 1988;82:1826-32. 2.Chopra S, Levell NJ, Cowley G, Gilkes JJ. Systemic corticosteroids in the phenytoin hypersensitivity syndrome. Br J Dermatol. 1996;134:1109-12. 3.Moss DM, Rudis M, Henderson SO. Cross-sensitivity and the anticonvulsant hypersensitivity syndrome. J Emerg Med. 1999;17:503-6. Item 34 Answer: (D) Seborrheic keratosis Educational Objective: Recognize the distinguishing features of benign and malignant pigmented tumors. Because of the public’s increasing awareness about skin cancer and the importance of early detection and treatment, patients frequently inquire about new or changing pigmented lesions. The lesion shown is a classic example of a seborrheic keratosis, which is a common benign tumor of keratinocytes. Seborrheic keratoses are 2-mm to 2-cm papules or plaques with an irregular, papillated surface covered with slight, greasy scale. Seborrheic keratoses are sharply demarcated from normal adjacent skin and have a stuck-on appearance. They are usually pigmented, but the degree of pigmentation may vary considerably from lesion to lesion and even within an individual lesion. Seborrheic keratoses are uncommon in childhood and increase in number with age. Although they are usually asymptomatic, lesions occasionally become inflamed, with accompanying tenderness and pruritus. Seborrheic keratoses must be differentiated from other pigmented cutaneous tumors. When there is doubt about the

diagnosis, a biopsy should be done to exclude malignancy. A congenital melanocytic nevus is present at birth, occurring in approximately 1% of newborns. Lesions vary in size from very small to large (covering a large area of the body). A congenital nevus has a smooth surface, occasionally with outgrowths of coarse terminal hairs. Malignant melanoma may be difficult to distinguish from a seborrheic keratosis. A melanoma is morphologically characterized by the ABCDs (asymmetry, border irregularity, color variegation, and a diameter > 6 mm). Most basal cell carcinomas contain little or no melanin; however, a pigmented variant exists. It can be distinguished from a seborrheic keratosis by its translucent, pearly border, with telangiectasias, central ulceration, and smooth surface. The pigment in a basal cell carcinoma tends to be stippled rather than evenly dispersed. Solar lentigo is a macular tan-brown lesion that develops in sun-exposed skin, but it usually is not confused with a seborrheic keratosis. Bibliography 1.Pariser RJ. Benign neoplasms of the skin. Med Clin North Am. 1998;82:1285-307. Item 35 Answer: (A) Skin biopsy for routine processing Educational Objective: Select the test most likely to confirm a diagnosis of cutaneous lupus erythematosus in a patient with nonscarring cutaneous disease. This patient has subacute cutaneous lupus erythematosus. This is a photosensitive eruption that may be preceded by a history of a polymorphous light eruption–like rash. Although some investigators in Scandinavia believe that polymorphous light eruption and subacute cutaneous lupus erythematosus can coexist, it is the belief of most authorities in the United States that the lesions of polymorphous light eruption that occurr prior to diagnosis of subacute cutaneous lupus erythematosus are probably forerunners of the condition, rather than a separate condition. Subacute cutaneous lupus erythematosus is best diagnosed by the presence of an interface dermatitis on routine histologic study. The patient may have a positive result on direct immunofluorescence, but this test may be falsely negative in about 40% of patients with subacute cutaneous lupus erythematosus and may be falsely positive when exposed skin is tested. Antinuclear antibodies are frequently abnormal in patients with subacute cutaneous lupus erythematosus (> 90%), but the presence of a positive test in a 48-year-old woman can also be false-positive. The anti-Ro (SS-A) antibody has been closely linked to subacute cutaneous lupus erythematosus, but it may occur in other conditions, including Sj ِgren’s syndrome and rheumatoid arthritis, and even in some normal individuals. The presence of a positive test in a patient such as the one presented in this case would be highly suggestive, but often on a single testing, the frequency of positivity is only 40% to 50%. Some patients with subacute cutaneous lupus erythematosus will also have Sj ِgren’s syndrome, but a Schirmer’s test is not part of the routine evaluation and would not be helpful diagnostically. Bibliography 1.Sontheimer RD. Skin disease in lupus erythematosus. Lupus. 1997;6:75-217. 2.Lee LA, Farris AD. Photosensitivity diseases: cutaneous lupus erythematosus. J Investig Dermatol Symp Proc. 1999;4:73-8. Item 36 Answer: (D) Trimethoprim-sulfamethoxazole Educational Objective: Recognize the risk involved in the use of low-dose weekly methotrexate therapy for the treatment of severe psoriasis. Patients treated with weekly low-dose pulse methotrexate therapy can have dramatic improvement in their quality of life in terms of both their psoriatic arthritis, if it is present, and their extensive psoriatic cutaneous lesions. Close laboratory monitoring is required, and vigilance must be continuous. Sudden changes in renal function can produce dramatic methotrexate toxicity caused by reduced renal clearance of the drug. A number of drugs can have a similar effect. Trimethoprimsulfamethoxazole causes a particularly severe drug interaction, with potentially dramatic elevation of blood levels of methotrexate. All nonsteroidal anti-inflammatory drugs (NSAIDs) are notorious for both reducing renal clearance of methotrexate and unbinding methotrexate from its carrier protein. However, this problem usually becomes a crisis with NSAIDs only if renal function deteriorates for some other reason,

whereas it is a problem with trimethoprim-sulfamethoxazole even in patients with normal renal function. Itraconazole has been associated with rare toxic hepatitis and can even produce neutropenia. However, it would not predispose patients to methotrexate toxicity, which occurred in this patient. Prednisone therapy is relatively contraindicated in patients with psoriasis because of the risk of rebound after taper, but it would not account for the toxicity in this patient. Propranolol and other beta-blockers can exacerbate psoriasis, but again, they would not predispose patients to the hepatic, hematologic, or gastrointestinal toxicity of methotrexate. Minocycline can be associated with nausea. It produces pancytopenia and liver function test abnormalities only very rarely and would not account for the methotrexate toxicity in this patient. Bibliography 1.Greaves MW, Weinstein GD. Treatment of psoriasis. N Engl J Med. 1995;332:581-8. 2.Thomas DR, Dover JS, Camp RD. Pancytopenia induced by the interaction between methotrexate and trimethoprim-sulfameth-oxazole. J Am Acad Dermatol. 1987;17:1055-6.

Item 37 Answer: (D) Treat individual lesions with liquid nitrogen, followed by a course of topical 5-FU cream. Educational Objective: Select the appropriate management for patients with actinic keratoses. Because actinic keratoses represent an early stage in the biologic continuum to squamous cell carcinoma, dermatologists believe that actinic keratoses are not trivial or cosmetic lesions and should be treated to prevent the development of squamous cell carcinoma. In addition, actinic keratoses may be painful or become irritated and bleed. The preferred treatment is destruction of the lesions by liquid nitrogen (–195 °C), either by direct application with a cotton-tipped applicator or by spray with a special spray cannister. This freezes the tissue to – 50 °C and destroys the cells. The technique involves frosting the lesion and the immediate surrounding millimeter of normal skin for several seconds. Prolonged freezing, such as that used for treating warts, is not necessary. Blister formation should be avoided. The frozen area immediately becomes reddened and stings. This lasts for about a day. Subsequently, the actinic keratosis crusts and disappears, leaving a well-healed area. Because the actinic keratosis process is totally intraepidermal, scarring is avoided if the proper amount of liquid nitrogen is applied. Cryosurgery is the treatment of choice for individual lesions; some thicker lesions may require retreatment. In many patients, it suffices as a monotherapy. However, when there are many lesions, as in this patient, combination therapy with liquid nitrogen initially for discrete lesions and individual large or thick actinic keratoses, followed by a course of topical 5-fluorouracil (5-FU) for about 2 weeks, is often the best course of action. Intralesional 5-FU has been used successfully for the treatment of keratoacanthoma and recalcitrant warts, but not actinic keratoses. Photodynamic therapy with a topical photosensitizer (aminolevulinic acid) that binds preferentially with the abnormal actinic keratosis cells combined with laser light has recently been approved by the Food and Drug Administration, but it is still very new and requires sophisticated equipment. Topical 5-FU would also be a good monotherapy, but it has significant drawbacks. It is a potent contact irritant in most patients, and about 10% of patients develop a contact hypersensitivity reaction, similar to the reaction caused by poison ivy. Both conditions translate to an increased incidence of local irritation, redness, swelling, weeping, oozing, crusting, pain, and tenderness. Many patients do not complete a course of treatment, and compliance is a problem. To avoid these side effects, attempts to vary the dosing with intermittent pulse treatment (three times a week, 1 week out of 4) or combining 5-FU with topical glucocorticoids (such as hydrocortisone or triamcinolone) is often helpful. Bibliography 1.Feldman SR, Fleischer AB Jr, Williford PM, Jorizzo JL. Destructive procedures are the standard of care for treatment of actinic keratoses. J Am Acad Dermatol. 1999;40:43-7.

2.Epstein E. Does intermittent “pulse” topical 5-fluorouracil therapy allow destruction of actinic keratoses without significant inflammation? J Am Acad Dermatol. 1998;38:77-80. 3.Abadir DM. Combination of topical 5-fluorouracil with cryotherapy for treatment of actinic keratoses. J Dermatol Surg Oncol. 1983;9:403-4.

4.Kurwa HA, Yong-Gee SA, Seed PT, Markey AC, Barlow RJ. A randomized paired comparison of photodynamic therapy and topical 5-fluorouracil in the treatment of actinic keratoses. J Am Acad Dermatol. 1999;41(Pt 1):414-8. Item 38 Answer: (C) Hydrochlorothiazide Educational Objective: Identify hydrochlorothiazide as a cause of subacute cutaneous lupus erythematosus. This patient has subacute cutaneous lupus erythematosus that might have been induced by her use of hydrochlorothiazide. Although she has adult acne and is taking minocycline, skin lesions rarely accompany the lupus-like disease that is associated with minocycline. Hydralazine also has been associated with drug-induced systemic lupus erythematosus, but the clinical manifestation is usually a serositis, not a skin rash. Both the minocycline-induced disease and hydralazine-induced disease have been associated with antihistone antibodies, and the minocycline-induced disease has been linked to a positive pANCA. The role of estrogen in patients with systemic lupus erythematosus is not clear, but it has not been linked to the development of drug-induced cutaneous lupus erythematosus. Etidronate has not been linked to drug-induced systemic lupus erythematosus or to drug-induced cutaneous lupus erythematosus. Bibliography 1.Reed BR, Huff JC, Jones SK, Orton PW, Lee LA, Norris DA. Subacute cutaneous lupus erythematosus associated with hydrochlorothiazide therapy. Ann Intern Med. 1985;103:4951. 2.Crowson AN, Magro CM. Subacute cutaneous lupus erythematosus arising in the setting of calcium channel blocker therapy. Hum Pathol. 1997;28:67-73. 3.Sturkenboom MC, Meier CR, Jick H, Stricker BH. Minocycline and lupuslike syndrome in acne patients. Arch Intern Med. 1999;159:493-7. 4.Pramatarov KD. Drug-induced lupus erythematosus. Clin Dermatol. 1998;16:367-77. Item 39 Answer: (D) Thalidomide Educational Objective: Select an appropriate therapy for a patient with refractory cutaneous lupus erythematosus. Recently, thalidomide has become available for off-label use in patients who have refractory cutaneous lupus erythematosus. Thalidomide is effective in relatively low doses and may be used over a long period with minimal toxicity. Concern for the possible development of neuropathy should be discussed with the patient prior to therapy, but the use of nerve conduction studies before and during therapy is not clear. Because this patient is postmenopausal, a pregnancy prevention program is not needed, but the company that supplies thalidomide has designed a program for its safe use. Although prednisone is useful for the treatment of systemic lupus erythematosus, its effectiveness for cutaneous lesions has been questioned and it has many side effects, making its use less than desirable for patients who require long-term therapy. Carbamazepine has not been reported to be effective in lupus erythematosus. Although both intravenous immunoglobulin and mycophenolate mofetil have been reported to be beneficial to some patients, their costs are too expensive for their selection at this time. Bibliography 1.Duong DJ, Spigel GT, Moxley RT 3rd, Gaspari AA. American experience with lowdose thalidomide therapy for severe cutaneous lupus erythematosus. Arch Dermatol. 1999;135:1079-87. 2.Callen JP. Management of antimalarial-refractory cutaneous lupus erythematosus. Lupus. 1997;6:203-8.

Item 40

Answer: (C) Pemphigus vulgaris or paraneoplastic pemphigus with antibodies against desmoglein 3 or desmoglein 1 and 3 and the plakin proteins, respectively Educational Objective: Recognize the clinical findings of pemphigus and its variants and their associated antigens. This older patient has an intraepidermal blistering disorder, which is determined clinically by extension of an intact blister with application of pressure. The most likely diagnosis is pemphigus vulgaris or paraneoplastic pemphigus, in which the antigens are desmoglein 3 or desmoglein 1 and 3 and the plakin proteins, respectively. Although lichen planus can sometimes present as a bullous eruption, its pathologic site is subepidermal as is that of bullous pemphigoid. A thiazide-induced reaction would not be expected to be associated with circulating autoimmune antibodies. Bibliography 1.Anhalt GJ. Making sense of antigens and antibodies in pemphigus. J Am Acad Dermatol. 1999;40(Pt 1):763-6. 2.Scott JE, Ahmed AR. The blistering diseases. Med Clin North Am. 1998;82:1239-83. 3.Bystryn JC, Steinman NM. The adjuvant therapy of pemphigus. An update. Arch Dermatol. 1996;132:203-12. Item 41 Answer: (C) Arrange for excision of the entire lesion. Educational Objective: Understand the risk of melanoma in congenital nevi. This patient most likely has a melanoma developing within a medium-sized congenital nevus. The gray hairs may or may not be related to the anaplastic process, but the nodule formation and notching of the medial border certainly are. With such a high level of suspicion, excision of the entire tumor is warranted. Removing the entire lesion not only insures that an appropriate histologic diagnosis can be made, but also facilitates identifying the Breslow thickness and Clark level invasion. The latter two parameters are the most important variables in predicting the risk of metastases. Full-thickness incisional biopsy has been advocated as a means of monitoring bumps and nodules that develop within large, unresectable congenital nevi. However, although a tangential biopsy of the nodule would identify melanoma, it would actually interfere with an assessment of tumor thickness and level of invasion. The changes in this lesion are dramatic, and observation or reassurance would be inappropriate management options for this patient. Lymphatic mapping in combination with sentinel node biopsy can be done, if warranted, at the time of wide local excision (that is, after a diagnosis of melanoma is established and prognostic variables have been obtained). Congenital nevi are present in approximately 1 in 100 babies born in the United States. Most of these congenital nevi are small, but about 1 in 20,000 are large or giant. Congenital nevi have been arbitrarily divided into small (< 1.5 cm), medium-sized (1.5 to 19.9 cm), and large (> 20 cm), according to the size they will attain in adulthood. The last are also referred to as hairy, giant, or bathing trunk nevi, depending on their location and configuration. The large nevi are the most worrisome. Besides being cosmetically disfiguring, they have a 5% to 10% chance of developing melanoma. This is a concern during childhood, because more than 50% of melanomas develop before 5 years of age, whereas 70% develop by 10 years of age. Intervention should be done as early as possible. The risk of melanoma in small and medium-sized tumors is dramatically different, being no higher than that in the general population. Although there are no lifetime prospective studies, the incidence of melanoma developing in the larger medium-sized nevi is very low (less than 4%), and when melanoma does occur, it is always in adulthood, as in this patient. Therefore, unless small or medium-sized lesions have unusual features (for example, a notched border or nodules), are in difficult-to-observe areas, such as the scalp, or are cosmetically unappealing, lifetime observation is adequate. However, when significant change does occur, such as nodule formation, notching of the border, areas of regression, or symptoms such as bleeding, pain, or itching, biopsy is required. Bibliography 1.Sahin S, Levin L, Kopf AW, Rao BK, Triola M, Koenig K, et al. Risk of melanoma in mediumsized congenital melanocytic nevi: a follow-up study. J Am Acad Dermatol. 1998;39:428-33.

2.Swerdlow AJ, English JS, Qiao Z. The risk of melanoma in patients with congenital nevi: a

cohort study. J Am Acad Dermatol. 1995;32:595-9. 3.Egan CL, Oliveria SA, Elenitsas R, Hanson J, Halpern AC. Cutaneous melanoma risk and phenotypic changes in large congenital nevi: a follow-up study of 46 patients. J Am Acad Dermatol. 1998;39:923-32. 4.Marghoob AA, Schoenbach SP, Kopf AW, Orlow SJ, Nossa R, Bart RS. Large congenital melanocytic nevi and the risk for the development of malignant melanoma. A prospective study. Arch Dermatol. 1996;132:170-5. Item 42 Answer: (A) Lichen planus Educational Objective: Recognize lichen planus–like eruptions as a manifestation of graftversus-host disease. The cutaneous eruption associated with chronic graft-versus-host disease following allogeneic bone marrow transplantation most closely resembles lichen planus. It is of interest that lichen planus is a prototypic disease manifesting histologic apoptosis. Lymphocytes can be seen adjacent to dying keratinocytes, as would be expected in a graft-versus-host–like reaction. Pityriasis rosea–like eruptions can occur in patients with a hypersensitivity to medications (most typically gold salts) but are not a manifestation of graft-versus-host disease. Although some lichen planus–like eruptions may resemble psoriasis clinically, in that both are papulosquamous diseases, the dermatologic reactions in graft-versus-host disease resemble lichen planus–like eruptions, both clinically and histopathologically. Atopic dermatitis is not a manifestation of chronic graft-versus-host disease. Tinea corporis, like lichen planus, pityriasis rosea, and psoriasis, is a papulosquamous eruption with lesions beginning as papules with fine scale, but it should not be confused with the lichen planus–like eruptions of graft-versus-host disease. It can be excluded by examination of potassium hydroxide preparations and by fungal culture of scale obtained from a lesion. Bibliography 1.Boyd AS, Neldner KH. Lichen planus. J Am Acad Dermatol. 1991;25:593-619. 2.Cribier B, Frances C, Chosidow O. Treatment of lichen planus. An evidence-based medicine analysis of efficacy. Arch Dermatol. 1998;134:1521-30. Item 43 Answer: (D) Doxycycline Educational Objective: Recognize the risks associated with using tanning beds in patients taking photosensitizing drugs. Patients are often warned to avoid sun exposure when they are prescribed a photosensitizing medication. Photosensitive reactions to drugs are usually related to exposure to ultraviolet A radiation (320 to 400 nm). The enhanced percentage of ultraviolet A radiation, as opposed to shorter-wavelength ultraviolet B radiation, in tanning beds actually makes that exposure of even greater risk than sunlight for producing photosensitive reactions to medications. Doxycycline, a ringed molecule in the tetracycline family, has often been implicated as a photosensitizing drug. Colchicine has not produced photosensitive reactions. Hydroxychloroquine is often used to treat photosensitive diseases, such as systemic lupus erythematosus, but it is not a photosensitizer. Beta-carotene has been reported to benefit patients with photosensitivity diseases such as polymorphic light eruption and erythropoietic protoporphyria, but it does not produce photosensitive reactions itself. Azathioprine is an immunosuppressive medication that has been used to treat patients with photosensitivity diseases such as systemic lupus erythematosus and chronic actinic dermatitis, but it does not by itself produce photosensitive reactions. Bibliography 1.Miranda MA. Assessment of the phototoxicity risk of new drugs. Arch Toxicol Suppl. 1997;19:249-58.

Item 44

Answer: (C) Cholinergic urticaria Educational Objective: Diagnose cholinergic urticaria. This patient has symptoms of cholinergic urticaria induced by heat, activity, and emotional stress. Not uncommonly, patients with active urticaria also demonstrate dermographism (immediate wheal and flare reaction with trauma to the skin), as is the case in this patient. This is different from pressure-induced urticaria, in which a painful whealing reaction occurs hours after local skin pressure or trauma. Cholinergic urticaria is often difficult to completely eliminate, but it can be effectively managed by the use of antihistamines and by having the patient avoid excessive heat, physical activity, and stress. Antihistamines are usually effective in controlling the signs and symptoms of dermographism. Bibliography 1.Zuberbier T, Althaus C, Chantraine-Hess S, Czarnetzki BM. Prevalence of cholinergic urticaria in young adults. J Am Acad Dermatol. 1994;31:978-81. 2.Humphreys F, Hunter JA. The characteristics of urticaria in 390 patients. Br J Dermatol. 1998;138:635-8. Item 45 Answer: (C) Rely primarily on protective clothing and hats. Educational Objective: Learn the components of a comprehensive “sun-protection” program. This professional golfer will be unable to totally avoid sun exposure. However, he can modify his behavior to limit his exposure. Scheduling tee times during tournaments is out of his control, but he can arrange practice rounds early or late in the day, when the sun is not overhead. Avoiding outdoor activities during the times of most intense ultraviolet radiation exposure, between 10:00 AM and 4:00 PM (depending on the time of year and latitude), reduces the total lifetime exposure significantly. High-risk behaviors such as suntanning or going to a tanning salon should be avoided. A “sun-savvy” attitude is also helpful, for example, walking on the shady side of the street, using a sun umbrella at the beach or when spending time at outdoor sporting events, and purchasing tickets on the shady side of the field for outdoor sporting events. This patient can stand in the shade or under an umbrella when his partners are on the putting green. Protective clothing is probably the best line of defense for this patient when sun avoidance is a problem. Clothing should limit exposure to high-risk portions of the exposed skin that are prone to photoaging and skin cancers. These include the scalp, ears, neck, forearms, and the V of the neck. In general, regular use of wide-brimmed hats and long-sleeved shirts and blouses help accomplish this goal. These articles of clothing and hats with wide brims or neckprotective extensions are readily available and fashionable enough for any outdoor activity. Sunscreen use can be added as the final strategy in a comprehensive sun-smart approach to life. Regular and proper application of high-potency and wide-spectrum sunscreens supplements the effects of sun avoidance and protective clothing. High-risk areas such as the nose, ears, scalp, and neck can be covered quickly and easily. The three strategies of sun avoidance, protective clothing, and sunscreens are important. Tailoring their proportional use to a specific lifestyle is critical. This patient must rely on protective clothing as a primary strategy, with sunscreen an important adjunct. Over a lifetime, paying attention to sun avoidance saves hours of cumulative ultraviolet radiation exposure. Tanning salons play no role in a sun-smart program. This patient cannot select where and when he wants to play. Bibliography 1.Guercio-Hauer C, Macfarlane DF, Deleo VA. Photodamage, photoaging and photoprotection of the skin. Am Fam Physician. 1994;50:327-32, 334. 2.Loescher LJ, Buller MK, Buller DB, Emerson J, Taylor AM. Public education projects in skin cancer. The evolution of skin cancer prevention education for children at a comprehensive cancer center. Cancer. 1995;75(2 Suppl):651-6.

Item 46

Answer: (C) Wear a 4-inch broad-brimmed cloth hat. Educational Objective: Appreciate the protective effect of wearing a hat. The 4-inch broad-brimmed hat is the best form of mobile shade and is effective in protecting the ears, neck, and face, including the nose. The only disadvantage is its ungainliness for some physical activities, such as swinging a golf club. The cap with flaps that drape over the ears and neck would be a good choice if it protected the cheeks and nose more effectively. It provides cover for all the areas at risk, except the nose, and provides shade for the eyes. The plain baseball cap does not cover the ears and neck, although it protects the scalp and forehead adequately. It is the most popular form of recreational headgear, but it does not protect the wearer from sunlight coming in from either side of the front brim. The fenestrated straw hat and other types of meshed hats are cooler, but they allow sunlight through the wide holes and give a false sense of protection. In general, clothing, in this case in the form of a hat, is better protection against ultraviolet radiation than sunscreens. Hats are the most important article of clothing when it comes to sun protection. For men, the ears are especially prone to overexposure. Eighty-five percent of skin cancers occur on the ears. If worn properly, a hat offers mobile shade for these areas. A 4-inch broad-brimmed hat is the most effective, because it provides the best protection for all the areas at risk, especially the temples, cheeks, and nose during the middle of the day, regardless of the person’s angle to the sun. It has been estimated that each inch of brim on a hat reduces the incidence of skin cancer by 10%. Bibliolgraphy 1.Diffey BL, Cheeseman J. Sun protection with hats. Br J Dermatol. 1992;127:10-2. Item 47 Answer: (B) Lichenification Educational Objective: Identify the clinical features of chronic eczematous eruptions. All types of eczematous eruptions are associated with histologic spongiosis, which represents edema within and around epidermal cells. Lichenification, however, is the characteristic clinical feature of chronic eczematous eruptions. This thickening of the outer layers of the skin is exacerbated by chronic rubbing. It manifests clinically as an exaggeration of skin markings with thickening, which is often accompanied by scale and dyspigmentation. Vesicles are a defining feature of acute eczematous eruptions. The Koebner phenomenon (the tendency of eruptions to spread into sites of trauma) is seen with several papulosquamous diseases, such as psoriasis and lichen planus, but it does not occur in chronic eczematous eruptions. Dermal edema occurs in many conditions but is particularly characteristic of urticarial eruptions. An annular or ringlike configuration is seen with many erythematous eruptions, especially erythema migrans seen in patients with Lyme disease, but it is not a feature of chronic eczematous eruptions. Bibliography 1.MacKenzie-Wood AR, Freeman S. Unclassified endogenous eczema. Contact Dermatitis. 1999;41:18-21. Item 48 Answer: (D) Squamous cell carcinoma Educational Objective: Recognize actinic keratoses as an early clinical stage in the progression of squamous cell carcinoma. There is now compelling epidemiologic, histopathologic, and molecular biologic evidence that actinic keratosis and squamous cell carcinoma are different stages of an oncologic continuum that ranges from dysplastic preclinical change, through actinic keratosis, to invasive squamous cell carcinoma, and, finally, to metastatic squamous cell carcinoma. It is accepted that ultraviolet radiation from sun exposure damages the DNA of the keratinocytes, which are the primary cell line of the skin. Damaged cells either die through apoptosis or survive with complete or faulty repair of the DNA. Cells that survive persist as mutations and lead to clonal expansion and clinical lesions recognized as actinic keratoses after they receive further exposure to ultraviolet radiation. Some actinic keratoses persist as an intermediate end point in the cancer spectrum, some regress, and others progress to invasive squamous cell

carcinoma. Approximately 6% to 10% of actinic keratoses progress to squamous cell carcinoma. Predicting which actinic keratoses will progress to squamous cell carcinoma is impossible clinically. Therefore, treatment of actinic keratoses is deemed important in the prevention of squamous cell carcinoma. Likewise, it is important to do a complete skin examination on patients with actinic keratoses to detect lesions that may have already progressed to or are suspicious for squamous cell carcinoma because they are larger or thicker than the other actinic keratoses. Actinic keratoses act as a dose meter, measuring the patient’s unique susceptibility to ultraviolet light radiation, as well as the cumulative ultraviolet light radiation exposure, and they identify patients at risk for basal cell carcinoma and, to a lesser extent, melanoma. Keratoacanthoma is another type of skin cancer related to squamous cell carcinoma. It is believed to be an abortive variety of squamous cell carcinoma, in which the tumor grows rapidly to a size of 1 to 2 cm over a 4- to 6-week period, remains quiescent for a short time, and then regresses over a period of 4 to 6 weeks. Bibliography 1.Dodson JM, DeSpain J, Hewett JE, Clark DP. Malignant potential of actinic keratoses and the controversy over treatment. A patient-oriented perspective. Arch Dermatol. 1991;127:1029-31. 2.Moon TE, Levine N, Cartmel B, Bangert JL, Rodney S, Dong Q, et al. Effect of retinol in preventing squamous cell skin cancer in moderate-risk subjects: a randomized, double-blind, controlled trial. Southwest Skin Cancer Prevention Study Group. Cancer Epidemiol Biomarkers Prev. 1997;6:949-56. 3.Salasche SJ. Epidemiology of actinic keratoses and squamous cell carcinoma. J Am Acad Dermatol. 2000;42(Pt 2):4-7. 4.Guenthner ST, Hurwitz RM, Buckel LJ, Gray HR. Cutaneous squamous cell carcinomas consistently show histologic evidence of in situ changes: a clinicopathologic correlation. J Am Acad Dermatol. 1999;41(Pt 1):443-8. 5.Ziegler A, Jonason AS, Leffell DJ, Simon JA, Sharma HW, Kimmelman J, et al. Sunburn and p53 in the onset of skin cancer. Nature. 1994;372:773-6. Item 49 Answer: (B) Avoid going to the tanning salon entirely. Educational Objective: Understand the futility and harmful effects of obtaining a “base tan” at a tanning salon. There is a popular misconception that because naturally dark-skinned people are less likely to sunburn or develop a melanoma, a “base tan” for light-complexioned people will provide the same protection. This is a fallacy at several levels. First, the process of obtaining a tan implies that damage has been done to the DNA of epidermal cells. Tanning is a defensive respone by the skin to ultraviolet radiation. Because it is epidermal cell injury that initiates the process of melanogenesis, there is no such thing as a healthy tan. Second, a tan is not very protective. A “base tan,” however obtained, may increase the sun protective factor (SPF) of fair skin from a baseline of 0.5 to only 3. A product that would offer an SPF of only 3 would hardly be worth applying to the skin, as it would only triple the amount of time someone could stay out in the sun without burning. For a fair-skinned redhead, this may mean an increase in protected time of only about a half hour. This is not commensurate with the widely presumed belief that the base tan allows for extra time in the sun without further protection. Finally, the indoor tanning industry, a $2 billion per year conglomerate, once claimed that tanning is harmless and healthy. Recently, the Federal Trade Commission ruled that the industry could no longer make health and safety claims, only claims for cosmetic enhancement. Furthermore, to decrease the time required on the tanning bed, tanning salon owners have added ultraviolet B wavelengths to complement the “safe” ultraviolet A rays. With this addition, burns have occurred, and the risk of skin cancers is increased. In addition, the tans produced are not protective, as lentigo-like lesions have been produced, and an increase in the incidence of nonmelanomatous skin cancer has been reported. Bibliography

1.Spencer JM, Amonette RA. Indoor tanning: risks, benefits, and future trends. J Am Acad Dermatol. 1995;33(Pt 1):288-98. 2.Spencer JM, Amonette R. Tanning beds and skin cancer: artificial sun + old sol = real risk. Clin Dermatol. 1998;16:487-501. 3.Westerdahl J, Olsson H, Masback A, Ingvar C, Jonsson N, Brandt L, et al. Use of sunbeds or sunlamps and malignant melanoma in southern Sweden. Am J Epidemiol. 1994;140:691-9.

4.Autier P, Dore JF, Lejeune F, Koelmel KF, Geffeler O, Hille P, et al. Cutaneous malignant melanoma and exposure to sunlamps or sunbeds: an EORTC multicenter case-control study in Belgium, France and Germany. EORTC Melanoma Cooperative Group. Int J Cancer. 1994;58:809-13. Item 50 Answer: (D) Refer her to a dermatologist for patch testing to a number of common antigens, including rubber additives. Educational Objective: Differentiate immediate latex allergic reactions from delayed reactions caused by rubber additives. This patient has skin changes of a chronic allergic (delayed) contact dermatitis. The diagnostic approach is to perform patch testing to rubber additives, which are the most common cause of this reaction. Radioallergosorbent (RAST) and skin-prick tests to latex antigens would test for the presence of IgE, which is not relevant to this patient’s reaction. A skin biopsy would not provide a specific answer to the cause of the reaction, and inhalation challenges for patients with IgE antibodies to latex could be fatal. Bibliography 1.Warshaw EM. Latex allergy. J Am Acad Dermatol. 1998;39:1-24. 98337150 2.Wyss M, Elsner P, Wuthrich B, Burg G. Allergic contact dermatitis from natural latex without contact urticaria. Contact Dermatitis. 1993;28:154-6. Item 51 Answer: (B) Sunscreens should be applied about a half hour before going into the sun. Educational Objective: Understand the important aspects of selecting and using sunscreens. Today’s refined sunscreens represent a major development in the battle against skin cancer and photoaging. There is now convincing evidence that they can help prevent the acute damage of sunburn and the aging effects of chronic sun exposure. Evidence also shows that sunscreens can reduce the incidence of actinic keratoses and nonmelanomatous skin cancers. To be effective, a sunscreen must be capable of blocking wavelengths of ultraviolet radiation across the entire solar spectrum and of sufficient sun protective factor (SPF) strength to prevent ultraviolet radiation from reaching the skin. Both the strength and width of the spectrum covered by a sunscreen must account for the combination of a patient’s response to the sun, the latitude where the patient lives, and the scope of the outdoor activities pursued. They must be used properly and in sufficient quantity to cover exposed areas. Many people do not use sunscreens in sufficient strength or quantity and do not reapply sunscreen during prolonged sun exposure, and they have a false sense of security. They will not adhere to sun avoidance techniques or wear protective clothing, and they stay out in the sun longer than they should. In these instances, the opposite of the desired goal will occur—more skin cancers will develop. Bibliography 1.McLean DI, Gallagher R. Sunscreens. Use and misuse. Dermatol Clin. 1998;16:219-26. 2.Diffey BL, Grice J. The influence of sunscreen type on photoprotection. Br J Dermatol. 1997;137:103-5. 3.Naylor MF, Boyd A, Smith DW, Cameron GS, Hubbard D, Neldner KH. High sun protection factor sunscreens in the suppression of actinic neoplasia. Arch Dermatol. 1995;131:170-5.

4.Thompson SC, Jolley D, Marks R. Reduction of solar keratoses by regular sunscreen use. N Engl J Med. 1993;329:1147-51. 5.Robinson JK, Rademaker AW. Sun protection by families at the beach. Arch Pediatr Adolesc Med. 1998;152:466-70. Item 52

Answer: (B) Tightness of the weave of the fabric Educational Objective: Identify the properties of clothing fabric that add to or detract from its ability to protect against ultraviolet radiation. Clothing is obviously an important element in sun protection. Simple maneuvers such as wearing long-sleeved shirts rather than short-sleeved shirts significantly reduce ultraviolet radiation exposure to the forearms. Similarly, buttoning shirts up to the top button and turning up the collar do the same for the anterior chest and neck. Not all fabrics are equally effective in blocking ultraviolet radiation. For example, widely available tee shirts may offer protection (similar to the sun protection factor [SPF] of a sunscreen), ranging from an SPF as low as 6 to 8 to one as high as 16 to 20. Several variables explain these differences, including the type of fiber used in the fabric, how the fabric is designed and woven, and the color of the fabric. The most important sun-protective property of a cloth is how tightly it is woven. The tiny holes created by the weave or knit allow some sunlight to pass through. The tighter this weave, the smaller the holes and the more protection it affords. Holding a garment up to light gives a good indication of the protection the garment would provide from the sun. Knit polo shirts are the least effective sun-protective shirts. The shirt’s hole size is also affected by how tightly the garment is stretched over the skin. Snugly fitting clothes stretch over the shoulders and upper back, widening the interstices of the weave and lowering the SPF. The difference in SPF of the tail of a shirt may be double the portion stretched over the shoulders. Sweating soaks a garment and greatly reduces the SPF of a fabric. All the holes become filled with water, which acts as a magnifying glass, allowing transmission of the sun’s rays. However, washing a tee shirt for the first time in hot water significantly increases the SPF, because the garment shrinks and the intervening holes become smaller. Other characteristics of clothing are also important in protecting the skin from the sun, especially the thickness or density of the material. With time and repeated wash and wear cycles, the fabric thins, and the SPF decreases. Some materials, such as cotton, contain lignins, which are ultraviolet radiation-absorbing pigments. Using bleach in the wash cycle can negate the protective effect of these lignins. However, some fluorescent whitening agents actually absorb long-wave ultraviolet A radiation and increase the SPF. Some manufacturers add ultraviolet-absorbing compounds during the manufacturing process to decrease photodegradation. These compounds, such as RayosanR, also increase the SPF of the garment. Lastly, the darker shades of some ultraviolet-absorbing dyes are more effective protective aids than the lighter shades of the same dye. Although wearing long-sleeved shirts and long pants makes sense, the challenge has been to design comfortable, loose-fitting, lightweight fabrics that provide adequate protection against ultraviolet radiation. The new boutique high-SPF clothing takes advantage of all these properties, but it is more expensive than routinely available articles of clothing. Bibliography 1.Adam J. Sun-protective clothing. J Cutan Med Surg. 1998;3:50-3. 2.Sayre RM, Hughes S. Sun protective apparel, advancements in sun protection. Skin Cancer l993;8:41-7. 3.Stanford DG, Georgouras KE, Pailthorpe MT. Sun protection by a summer-weight garment: the effect of washing and wearing. Med J Aust. 1995;162:422-5. 4.O’Quinn RP, Wagner RF Jr. Unusual patterns of chronic photodamage through clothing. Cutis.

1998;61:269-71. 5.Menter JM, Hollins TD, Sayre RM, Etemadi AA, Willis I, Hughes SN. Protection against UV photocarcinogenesis by fabric materials. J Am Acad Dermatol. 1994;31(Pt 1):711-6.

Item 53

Answer: (C) Laxity Educational Objective: Recognize skin changes attributable to intrinsic chronologic aging as opposed to skin changes related to injury. All of the skin changes listed except laxity are caused by photodamage. Two processes lead to cutaneous changes associated with aging. Extrinsic aging is related to environmental influences, such as ultraviolet radiation, smoking, wind, and chemical exposure. Persistent exposure to ultraviolet light is the most significant cause of extrinsic aging, resulting in skin coarseness, actinic/solar keratoses, lentigines, fine lines, and telangiectasias. Intrinsic chronologic aging is largely genetically determined. Intrinsic changes include the natural effects of gravity (sagging/laxity), furrow or expression lines, and atrophy of the dermis and subcutis that often leads to the gaunt appearance of old age. Daily use of a broad-spectrum sunscreen with a high sun protective factor (SPF) or a sunscreen that combines a ultraviolet B and ultraviolet A block helps prevent ultraviolet damage and premature extrinsic photoaging. Bibliography 1.Taylor CR, Sober AJ. Sun exposure and skin disease. Annu Rev Med. 1996;47:181-91. 2.Kaminester LH. Current concepts. Photoprotection. Arch Fam Med. 1996;5:289-95. 3.Benedetto AV. The environment and skin aging. Clin Dermatol. 1998;16:129-39. 4.Castanet J, Ortonne JP. Pigmentary changes in aged and photoaged skin. Arch Dermatol. 1997;133:1296-9.